Friday, March 4, 2011

Scientism roundup

In several recent posts we have dealt at least indirectly with scientism, the view that the only real knowledge is scientific knowledge.  Scientism is an illusion, a bizarre fantasy that makes of science something it can never be.  Seemingly the paradigm of rationality, it is in fact incoherent, incapable in principle of being defended in a way consistent with its own epistemological scruples.  It should go without saying that this in no way entails any criticism of science itself.  For a man to acknowledge that there are many beautiful women in the world does not entail that he doesn’t think his own wife or girlfriend is beautiful.  Similarly, to say that there are entirely rational and objective sources of knowledge other than science does not commit one to denying that science is a source of knowledge.  Those who cannot see this are doubly deluded – like a vain and paranoid wife or girlfriend who thinks all women are far less attractive than she is and regards any suggestion to the contrary as a denial of her own beauty.  Worse, like an already beautiful woman whose vanity leads her to destroy her beauty in the attempt to enhance it through plastic surgery, scientism threatens to distort and corrupt science precisely by exaggerating its significance.

I have examined this topic at length in various articles and blog posts (as well as in The Last Superstition, which could be read as a book-length reply to scientism).  For readers who might be interested, I thought it would be useful to gather the relevant links together in one post (to which I will from time to time add further links to any future pieces related to the subject).

For a general critique of scientism, see my two-part article from Public Discourse:



For discussion of the differences between the scientific and philosophical approaches to the study of reality in general and of the natural world in particular, see:


For discussion of some specific errors connected with scientism – such as the claim made by some scientists that science has refuted certain traditional philosophical theses, or the muddleheaded assumption that “laws of nature” by themselves can explain anything, or the notion that science alone deals with “facts” – see these posts: 

“Why are (some) physicists so bad at philosophy?” 

"Coyne on intentionality" 

“The early Wittgenstein on scientism”


Stop it, you’re killing me! [on eliminativism vis-à-vis life]




 

For reviews of some recent books claiming to offer scientific answers to traditional philosophical questions, see: 

For a reply to the claim that modern biology has refuted the doctrine of original sin, see:

For discussion of the shrill dogmatism and circular reasoning that the scientism of the “New Atheist” writers leads them into, see:

“The New Philistinism” (an article I wrote for The American)


"A clue for Jerry Coyne"

"Tom and Jerry"

The pointlessness of Jerry Coyne


"So you think you understand the cosmological argument?"

"Grow up or shut up"

"Rosenhouse redux"

"Argumentum ad Himmlerum"

"Eric MacDonald's assisted intellectual suicide"

"A final word on Eric MacDonald"

"Addendum [on MacDonald]"

Scientism is related to naturalism, the view that serious philosophy is continuous with natural science and that all genuine philosophical problems can be solved at least indirectly by further research in natural science.  Naturalism is ultimately as indefensible as scientism is.  For some general criticisms of naturalism, see:



The philosophical assumptions about what counts as “natural” that inform naturalism and scientism have a tendency to lead those views in an “eliminativist” direction – that is to say, in the direction of denying that common sense features of the world such as the ordinary objects of our experience, moral values, and indeed the human mind itself, really exist at all.  Paul Churchland and Alexander Rosenberg are two philosophers well known for acknowledging and defending the eliminativist implications of naturalism.  But a thoroughgoing eliminativism is also incoherent.  (And though eliminativists like to pretend otherwise, the reasons have to do with far more than merely accusing them of “believing that there are no beliefs,” a specific incoherence that the eliminativist can of course easily sidestep by simply avoiding the word “belief.”)  We had reason some time back to consider Rosenberg’s version of naturalism and its eliminativist implications, and criticized it in a series of posts:




Rosenberg would later develop his views at greater length in his book The Atheist's Guide to Reality, which I examined in detail in a series of blog posts and in a review of the book in First Things:



Scientism and naturalism derive whatever plausibility they have from the assumption that almost everything has by now been explained by science in purely materialist terms, so that it is implausible to suggest that morality, the human mind, and other philosophically puzzling phenomena will not yield too to materialistic or naturalistic explanation.  But the idea that “everything else has been explained in materialist terms” is itself an illusion, based on sheer metaphysical sleight of hand.  Indeed, materialism rests on a conception of matter that makes a completely materialist account of the world in principle impossible, at least given the incoherence of eliminativism.  I explain why in:



Thomas Nagel's book Mind and Cosmos generated an enormous controversy over how much modern science has in fact explained or can explain.  I responded to Nagel's critics in a series of posts collected here:

Mind and Cosmos roundup

Conceptual confusion, unexamined prejudice, and ignorance of what non-materialist philosophers have actually said are common features of materialist and naturalist arguments about the mind.  I discuss these tendencies in:




I address these problems as they arise in the work of specific thinkers like Churchland, Frank Jackson, and Daniel Stoljar in some further posts:







Finally, it must be emphasized that to reject scientism and naturalism is by no means to endorse the claims of “Intelligent Design” theorists.  On the contrary, the chief problem with ID is precisely that it reflects something like a “scientistic” prejudice insofar as it assumes that reasoning from the order of the universe to the existence of an ordering intelligence involves a kind of “scientific theorizing.”  From an Aristotelian-Thomistic point of view, natural theology properly begins, not from this or that contingent empirical result of science, but rather from the deeper metaphysical truths that any possible empirical theory must presuppose – the distinction between actuality and potentiality, the reality of immanent final causality as a necessary concomitant of efficient causality, and other theses of the branch of metaphysics known as “philosophy of nature.”  I have criticized ID in a number of posts, links to which can be found here:

161 comments:

  1. Thank you for the useful summary!

    ReplyDelete
  2. Oh wow...my mind has just been Fesered!

    ReplyDelete
  3. Is it Dr. Feser? I forget.

    Why not write on Islam for a change? Atheism and secularism might be big issues in the West for theists/Christians/Catholic, but there is, at least, a lot of dialogue between the two groups. With Islam, though, there is absolutely no dialogue.

    ReplyDelete
  4. I actually would enjoy seeing some of that myself. New Atheism just doesn't have much 'there' intellectually, and I think Ed's done a lot on more grounded atheism and materialism in the past. At the very least I'd find critiques and commentary on other religious views of God (Islamic, Hindu, mormon, etc) from a Thomist perspective rivetting.

    But hey, that's me.

    ReplyDelete
  5. All in good time. Meanwhile, there is my old Tech Central Station article on Islam:

    http://www.ideasinactiontv.com/tcs_daily/2003/12/does-islam-need-a-luther-or-a-pope.html

    ReplyDelete
  6. Hey, thanks. Never saw this one.

    ReplyDelete
  7. Crude, just browse the "Articles" archive on my main website. It's a gold mine!

    http://www.edwardfeser.com

    ReplyDelete
  8. Dr. Feser, you write a lot about subjects that are easily refutable (i.e. scientism). Is there something you consider a more difficult objections to classical theism, aristotleian-thomism etc, and would you write about that?

    ReplyDelete
  9. Thanks for your great posts, Dr. Feser!

    By the way, somewhere in your book The Last Superstition you claim that Cartesian dualism violates "the laws of physics" because of the conservation of energy. I wonder if you have checked a recent book called The Soul Hypotheis, edited by Baker and Goetz (Continuum, 2011). In chapter 5 Robin Collins shows that the conservation of energy objection against interactionism is actually based on old physics, and current physics doesn´t support it at all, because energy is not conserved always in GR or Quantum mechanics. Maybe you want to change your views about this being a problem to Cartesian dualism, then?

    There´s a positive review of the book in Notre Dame Philosophical Reviews.

    ReplyDelete
  10. Just out of curiosity, how does postulating a soul (i.e. form) in the body (i.e. matter) explain first-person conscious awareness? I've read this explanation many times, but haven't been able to understand it. Can anyone help me out?

    ReplyDelete
  11. you write a lot about subjects that are easily refutable (i.e. scientism).

    What an odd thing to say. I've written two books (The Last Superstition and Aquinas) which, if their arguments succeed, refute atheism in general and not just the "New Atheism." (Certainly they deal with atheist objections in general, not just the feeble sort presented by Dawkins and Co.) In those books and elsewhere I also defend not just dualism, but A-T hylemorphic dualism specifically, and traditional natural law theory. Here on the blog I've gone after naturalism in general, materialism in philosophy of mind, new natural law theory, "Intelligent Design" theory, and have defended A-T metaphysics, the traditional natural law view of lying and just war, and who knows what else.

    Apparently that was all just low-hanging fruit! Who knew?

    ReplyDelete
  12. Dr. F,

    Thanks so much for collecting your thoughts on these matters. This post was very helpful and has already been reposted, facebooked, and sent personally to some friends. Keep up the good work.

    ReplyDelete
  13. Suggestion. Science is knowledge of a thing thru it's causes.

    If that is true, we have to distinguish between the types of sciences, since their objects (Thing studied) are different.

    Here are my suggestions for a shorthand to avoid confusion.

    p-science - Physical sciences
    m-science - Math
    met-science - Metaphysics.

    In our day and age many use the term science to only refer to the physical sciences. This annoys me to know end. I keep correcting people on this:

    (If you insist on questioning & correcting people like I do, you will have few friends)..
    I am also annoyed to no end, when I go to the bookstore and under Metaphysics, there are books on tarot cards etc.. GRRRRRRRRRRR

    It's hard to be serous about philosophy...

    ReplyDelete
  14. sorry, I mean "no end" in posting above

    ReplyDelete
  15. I was thinking something like, that perhaps someone had made a tough response to you and you thought the response to that would be too technical for us silly laypeople. I have TLS left in my queue of those, but the queue is moving too slowly :<

    ReplyDelete
  16. Eman: no, not that.. I am tired of the haughty people who think the p-sciences are the only way to obtain knowledge, and that philosophy is just baseless stream of cosciousness babble..I am not referring to people in general.
    (If u knew me, you'd know I can seem quite annoying.)

    ReplyDelete
  17. Scientism, atheism (new or not), and similar views are *not* easily refutable, and Ed goes into great detail about why this is the case, in The Last Superstition, before embarking on precisely that task.

    I am probably the slowest of his readers, but even I get that point, which was made abundantly clear quite early in the book, ad nauseam. The only reason I am not issuing substantial criticisms at this point is because I am first taking the trouble to actually *read* both the book and the blog thoroughly.

    Blogger needs to have, under each comment, the following clickable objects:

    "Hide this comment"
    "Hide all comments by this user"

    ReplyDelete
  18. "Scientism, atheism (new or not), and similar views are *not* easily refutable"

    You sure? I think the attitude that accompanies those two views are hard to shake out of a person, but scientism, at least, is as easy to refute as Sola Scriptura.

    ReplyDelete
  19. Sometimes a refutation within those topics is easy, even by Ed's view. See the end of "Rosenberg responds to his critics". I'm not sure it's right to speak of "refuting" the NAs, because that implies NAs have something substantial to say most of the time about topics they aren't mangling in the process.

    ReplyDelete
  20. I'm loving all the links, Prof. Feser. You should keep a mini-bibliography section for certain topics that you discuss. :)

    ReplyDelete
  21. To be honest, I greatly enjoy reading Dr. Feser's attacks on naturalism/materialism. I think he's tackled quite a few subjects on this blog and I don't see how people can really complain. Keep it up, I say!

    ReplyDelete
  22. I am not complaining. I just want to see more on Islam - there is very little apologetic/philosophical content on Islam that is PhD-level.

    ReplyDelete
  23. Maybe I’m missing something, but science does not “presuppose” the existence of an external world, causal regularities, and the power of human inquiry to discover truths about the world. These are all things that all human beings experience as true, except for those with psychiatric or neurological diseases. I do not assume that an external world exists. I experience an external world. I do not assume that there are causal regularities. I experience that there are causal regularities. I do not assume that human beings can uncover truths about the world. Human beings do, in fact, uncover truths about the world. These are things that are actually occurring, and not just hypothetical postulates.

    Scientific inquiry begins with our everyday experience of an external world that exhibits regularity. Within this world, when we have a question, then we engage in common sense inquiry. For example, if I hear drops on my window, and I wonder if it is raining, then if I want to know whether it is raining, then I get up and look out the window to see. Maybe the drops hitting my window are just the sprinkler? Science begins with this common sense form of inquiry, and then extends the reach of our senses by using technology (e.g. microscopes, telescopes, etc.) and by using a rigorous methodology to rule out confounding factors that may be distorting our perception of an event or causal relationship. It recognizes that human beings are fallible and prone to error, and thus must take steps to correct or adjust for this possibility.

    And science does not have to get outside of itself to justify itself. It is just a more sophisticated and rigorous form of inquiry into the empirical world that has actually been enormously successful, because of the very attention it pays to what is essential to successful inquiry in general.

    So, you can run a test, and compare two groups of people, one who attempt to figure out the world by closely observing it, gathering data, analyzing that data to attempt to identify causal patterns, and to attempt to rule out any confounding factors that may be distorting their perception, and another group who does … something else (e.g. rolling dice, seeking the predictions of astrologers, or whatever). Then you can compare who has a better understanding of the world around them. I’m pretty sure that you will be more impressed by the former than the latter, and one did not have to leave one’s skin to view science from a view from nowhere to justify its validity.

    ReplyDelete
  24. dguller,

    Your first paragraph is a succinct statement of philosophical realism. Unfortunately, it is currently denied by many philosophers and scientists of the mind.

    Many scientists of the mind claim that we don't actually experience an external world, at least directly. We only experience an internal model of the world, one formed by the mind as it organizes the sensations bombarding our senses. For instance, you don't actually hear "drops on a window." What you hear are fluctuations in pressure on the ear, and what you see are photons incident on your retina. Down the line, the neurons in your brain organize these sensations into an interior experience of sound, and a visual experience of rain and windows, and associates the two into the integrated audiovisual experience of "drops on a window."

    What is the relationship between this interior experience of "drops on a window" and the actual reality out there that is the objective basis of the experience? Is this something science can determine? It doesn't seem like it is, since scientific data is necessarily human experience, and so only appears to us only after all the mental processing happens. There is no way to "leave our skin", as you put it, and compare our mental models to the objective reality on which they are based. This is an insurmountable conundrum that haunts the contemporary science/philosophy of the mind and is, in the end, destructive of science itself.

    So one of the philosophical ideas that science presupposes - at least if we want science to be about the real world and not just our interior models of the world - is philosophical realism: What I see and hear are not interior models of drops and windows, but the drops and windows themselves. And the only way to defend this position is philosophically, since as soon as science has occurred, it has already assumed an answer to the ontological status of drops and windows.

    ReplyDelete
  25. Thank you Dr Feser for all your hard work. I for one certainly enjoy this blog a great deal. Thank you.

    ReplyDelete
  26. I think atheists and materialists are the hardest to argue against because their philosophical position is simply negation. It's always hard to argue against a negative, just like debating the conspiracy theorist who doesn't believe the moon landings happened. In my agnostic phase, I never thought the choice was between anything except atheism and Catholicism. And I think the New Atheist types tacitly accept that by concentrating their fire so heavily on the Catholic Church. Until the liberalised decadent West collapses under the influx of Islam, I think apologists are right to concentrate on refuting atheists.

    ReplyDelete
  27. Until the liberalised decadent West collapses under the influx of Islam, I think apologists are right to concentrate on refuting atheists.

    For my part, I'm not asking Ed to refute Islam or anything else (I'd have trouble thinking of Ed as an apologist besides). Any more than I look to him to refute Platonism or neoplatonism, etc. In fact, I think Ed would probably have some positive things to say about these other faiths - points in common, points of disagreement, etc.

    As for people who are hard to argue against, I think you're right about 'negation'. But both atheists, and certainly materialists, put forth a positive program as well typically (or at least walk in with one assumed.) It's not atheists or materialists who are hard to engage, but anyone who does nothing but criticize while at the same time staying mum on their own beliefs. All offense, no defense. Even a Christian could manage that - just don't bring up Christianity.

    ReplyDelete
  28. Dmt117:

    >> Many scientists of the mind claim that we don't actually experience an external world, at least directly. We only experience an internal model of the world, one formed by the mind as it organizes the sensations bombarding our senses. For instance, you don't actually hear "drops on a window." What you hear are fluctuations in pressure on the ear, and what you see are photons incident on your retina. Down the line, the neurons in your brain organize these sensations into an interior experience of sound, and a visual experience of rain and windows, and associates the two into the integrated audiovisual experience of "drops on a window."

    That’s a good description. What I find most striking in that description is that our mind is inherently an embodied, embedded and extended entity. In other words, the internal model only makes sense in the presence of a brain that is intimately connected to a body that is intimately connected to an external environment, and that the internal model that is generated is the result of that total system. I think that is reasonable to say, and helps to explain how some of our experiences are actually false.

    >> What is the relationship between this interior experience of "drops on a window" and the actual reality out there that is the objective basis of the experience? Is this something science can determine? It doesn't seem like it is, since scientific data is necessarily human experience, and so only appears to us only after all the mental processing happens. There is no way to "leave our skin", as you put it, and compare our mental models to the objective reality on which they are based. This is an insurmountable conundrum that haunts the contemporary science/philosophy of the mind and is, in the end, destructive of science itself.

    I think it only haunts science if you assume that all we can experience is that internal model. Why assume that? Why not say that our experience is a part of a complex system of interacting components, and that it only makes sense within that system? After all, that is what we experience. It is only when you start using your imagination to remove these different components that philosophical dilemmas result. In other words, it is only when you start imagining a mind without a body, or independent of an external world, that you get into “insurmountable conundrums”, as you called them. Context is key here, and when you change the context, then you change the significance and meaning of the concepts involved.

    >> So one of the philosophical ideas that science presupposes - at least if we want science to be about the real world and not just our interior models of the world - is philosophical realism: What I see and hear are not interior models of drops and windows, but the drops and windows themselves. And the only way to defend this position is philosophically, since as soon as science has occurred, it has already assumed an answer to the ontological status of drops and windows.

    Here is the problem that I see with that line of thinking. The appearance-reality distinction is parasitic upon our experience in the world, just as all the important philosophical distinctions that apparently undermine empirical experience itself. For example, we have the experience of having something appear to be X, but actually turn out to be Y. That occurs within the world to our experience, and that is where the distinction comes from. If you want to say that all of our experience is just appearance, then you no longer have an appearance-reality distinction at all, because it is ALL appearance, and then you cannot even say that our experience is just appearance, because appearance only makes sense in contradiction to reality, at least in the philosophically relevant sense. And furthermore, it is based upon the erroneous line of reasoning that states that if I am occasionally wrong about my experience, then all my experience must be wrong, which is fallacious.

    ReplyDelete
  29. This comment has been removed by the author.

    ReplyDelete
  30. This is off-topic, but of interest.

    Philosopher Wes Morriston has just published (in the journal Religious Studies) a critique of Craig's moral argument for God's existence. You can read the paper here:

    http://spot.colorado.edu/~morristo/DoesGodGround.pdf

    Keep in mind that Morriston is a theist philosopher, not an atheist. However, he's critical of theistic arguments like the kalam and the moral argument (he's sympathetic to the teleological argument, however).

    My own opinion is that Morriston fails to refute the plausibility of Craig's argument. In any case, I look forward to read Craig's reply.

    It would be interesting and informative to know Dr.Feser's comments on Morriston's paper in a separate post, both from the perspective of Thomism and from the particular perspective of Craig's moral argument.

    Does Morriston's critique refute or undermine Craig's version of the moral argument?

    ReplyDelete
  31. dguller,

    "I think it only haunts science if you assume that all we can experience is that internal model. Why assume that? Why not say that our experience is a part of a complex system of interacting components, and that it only makes sense within that system? After all, that is what we experience."

    The scientists of the mind don't assume that all we can experience is that internal model. They think they have proven it through science. For instance, what way is there to experience "drops on a window" other than through the internal model of it?

    Remember, I do not myself subscribe to this position. I think it is ultimately self-defeating. I'm merely pointing out that any answer to is necessarily philosophical rather than scientific. This is why your response to me was in the form of philosophical argument rather than scientific experiment.

    ReplyDelete
  32. @dmt117 and dguller,

    What I don't understand is why anyone would think that it makes a difference to science, whether what (physical) science studies is an internal model or an external reality? The data remains the same. The theories work the same. The evidential support or disproof of a theory works the same.

    ReplyDelete
  33. Scott,

    Centuries ago that was called "saving the appearances." In any case, try telling a scientist his theories are only about his internal model and not about the real world... you'll find it makes a great deal of difference to him.

    ReplyDelete
  34. Finished The Last Superstition a few days ago, and I'm reading Philosophy of the Mind. Love the polemic! Keep it up.

    ReplyDelete
  35. Dmt117:

    >> The scientists of the mind don't assume that all we can experience is that internal model. They think they have proven it through science. For instance, what way is there to experience "drops on a window" other than through the internal model of it?

    Yes, but you are ignoring the context in which that experience occurs. I mean, scientists of the mind also have empirical evidence of neuroanatomical pathways from the retina, which receives electromagnetic input via cone and rod cells, which is converted into electrical signals through the optic nerve, to the occipital lobe, and so on, that explains where the sensory input even comes from. None of that is in the visual experience, but is present outside awareness. And yet our visual experience allowed us to perceive this underlying pathway as essential to the visual experience itself.

    So, there is more going on than just the subjective experience, and no neuroscientist ignores this wider context within which we experience anything at all. It does not make sense to just isolate a fragment of the total process, identify it as primary, and then ignore the rest, because the totality is where the sensory experience gets its sense from. That would be like building a house, standing in the second floor, and imagining that the floor would still stand without the walls supporting it. Sure, you can kind of imagine a floor suspended in the air, but would you really hire a builder to make such a floor? Similarly, our sensory experience is not just the activation of a particular area of the brain, but is embedded within a complex sequence of events running in parallel that intimately connects our conscious awareness with the world.

    ReplyDelete
  36. Scott:

    I wholeheartedly agree with dmt117 on this.

    ReplyDelete
  37. "Yes, but you are ignoring the context in which that experience occurs. I mean, scientists of the mind also have empirical evidence of neuroanatomical pathways from the retina, which receives electromagnetic input via cone and rod cells, which is converted into electrical signals through the optic nerve, to the occipital lobe, and so on..."

    This is just the point where scientists fail to see the implications of their philosophical premises. They think their empirical evidence somehow escapes the predicaments in which they place ordinary experience. But "empirical evidence of neuroanatomical pathways" is in exactly the same predicament as "drops on a window." Both are experienced only after being conditioned by the mental model.

    When the scientist puts on his lab coat, his "microscope", "CAT scanner" and "ammeter" are not magically given an ontological dignity that "drops on a window" lack. They are mental models just as much as the drops are. And everything experienced through them is necessarily conditioned by the mental model. The upshot is that "brains", "neurons", "optic nerves" and "pathways" are no more direct expressions of reality than are "drops on a window." If I am called on to doubt whether there are "drops on a window" in reality, I must also doubt whether there are "microscopes" and brains in reality.

    Plato's Cave is a good expression of this dilemma. The prisoners in the cave are tied to their chairs and can only see shadows on the cave wall. Now it doesn't matter whether they look at the shadows through microscopes or telescopes, or create brilliant theories connecting the causal relations between the shadows. Their theories are still about nothing more than the shadows, not the reality in the cave that produces the shadow.

    Similarly, if we suppose that "drops on a window" are just constructions our mind puts on raw sensory experience, then our knowledge can never get beyond those constructions, including scientific knowledge.

    ReplyDelete
  38. I'm going through Sham Harris's book now on The Moral Landscape. I don't know which is worse. The rank ignorance in this book or how so many atheists on the back endorse it.

    That includes Richard Dawkins who used to say he unthinkingly believed science can say nothing about morals. Now he unthinkingly believes Harris's claim.

    ReplyDelete
  39. Dmt117:

    >> This is just the point where scientists fail to see the implications of their philosophical premises. They think their empirical evidence somehow escapes the predicaments in which they place ordinary experience. But "empirical evidence of neuroanatomical pathways" is in exactly the same predicament as "drops on a window." Both are experienced only after being conditioned by the mental model.

    I totally disagree. You are assuming from the start that I am trapped behind a veil of appearances and can only experience a subjective mental state. The problem with this assumption is that there is no evidence for it, and it actually results in a number of insoluble paradoxes.

    I think that it is important to begin at the beginning. The majority of human beings, if not all of them, begin with their experiences of an external world. I mean, that is just a fact. Ask anyone. I have actually met individuals who experience the world as you claim everyone does. They suffer from derealization, which basically means that they disbelieve that the world is real, and there are observable functional impairment as a result of this disbelief. If everyone experienced the world in that way, then everyone would be unable to function. So, what you claim is the default state is actually an uncommon psychiatric condition that inhibits normal function, and is a deviation from the normal state of affairs for human beings.

    What this means is that your assumption is actually not primary, but secondary. It takes some external event, such as exposure to philosophical skepticism, to get the ball rolling to doubt the veracity of our perception. And what is skepticism? It is a series of what-if fantasies that asks us to imagine different state of affairs that are intended to challenge our intuitions. So, this secondary process is attempting to alter the primary one by offering reasons to doubt the latter. And what are these reasons? That if you can be wrong about some of what you perceive, then you must be wrong about ALL of what you perceive, which is fallacious. That if you can imagine a state of affairs divorced from the world’s ordinary functioning, then you MUST take these states of affairs seriously, even though they get their very meaning from the world that they say is unreal, and thus effectively refute themselves.

    What does all this mean? That we all begin with naïve realism as primary, and that there are no good reasons to doubt it, which means that you can trust that your senses actually engage with an external world. Our knowledge of neuroscience simply provides us with information about how we do this, and we acquired this information from our generally reliable senses. There is no reason to think that we are trapped behind a veil of appearances in a state of solipsism. Otherwise, why is there not rank confusion between human beings? How is language even possible without a bedrock of stability in our environment? Our perceptions provide information about the world via well-known physical mechanisms, and even when they fail to provide accurate information is within the context of a part of the total physical system itself failing to function for natural reasons.

    ReplyDelete
  40. Dmt117:

    >> This is just the point where scientists fail to see the implications of their philosophical premises. They think their empirical evidence somehow escapes the predicaments in which they place ordinary experience. But "empirical evidence of neuroanatomical pathways" is in exactly the same predicament as "drops on a window." Both are experienced only after being conditioned by the mental model.

    I totally disagree. You are assuming from the start that I am trapped behind a veil of appearances and can only experience a subjective mental state. The problem with this assumption is that there is no evidence for it, and it actually results in a number of insoluble paradoxes.

    I think that it is important to begin at the beginning. The majority of human beings, if not all of them, begin with their experiences of an external world. I mean, that is just a fact. Ask anyone. I have actually met individuals who experience the world as you claim everyone does. They suffer from derealization, which basically means that they disbelieve that the world is real, and there are observable functional impairment as a result of this disbelief. If everyone experienced the world in that way, then everyone would be unable to function. So, what you claim is the default state is actually an uncommon psychiatric condition that inhibits normal function, and is a deviation from the normal state of affairs for human beings.

    What this means is that your assumption is actually not primary, but secondary. It takes some external event, such as exposure to philosophical skepticism, to get the ball rolling to doubt the veracity of our perception. And what is skepticism? It is a series of what-if fantasies that asks us to imagine different state of affairs that are intended to challenge our intuitions. So, this secondary process is attempting to alter the primary one by offering reasons to doubt the latter. And what are these reasons? That if you can be wrong about some of what you perceive, then you must be wrong about ALL of what you perceive, which is fallacious. That if you can imagine a state of affairs divorced from the world’s ordinary functioning, then you MUST take these states of affairs seriously, even though they get their very meaning from the world that they say is unreal, and thus effectively refute themselves.

    ReplyDelete
  41. Dmt117:

    What does all this mean? That we all begin with naïve realism as primary, and that there are no good reasons to doubt it, which means that you can trust that your senses actually engage with an external world. Our knowledge of neuroscience simply provides us with information about how we do this, and we acquired this information from our generally reliable senses. There is no reason to think that we are trapped behind a veil of appearances in a state of solipsism. Otherwise, why is there not rank confusion between human beings? How is language even possible without a bedrock of stability in our environment? Our perceptions provide information about the world via well-known physical mechanisms, and even when they fail to provide accurate information is within the context of a part of the total physical system itself failing to function for natural reasons.

    The question is whether philosophical skepticism has uncovered a deep truth that we must come to grips with or whether it is just an illusion that is playing a trick on the mind. When I go to see a magician, and he performs amazing feats, I do not run out to reject natural explanations, because I know that I am being tricked. This is especially true when I am told how the trick even happens to begin with. Or take visual illusions, such as lines that do not move appearing to move on a page. I do not conclude that there is movement without movement, which should lead to a radical revision in the laws of motion and movement. No, I conclude that it is illusory and unreal, and occurs by playing upon vulnerabilities in how I process visual information. I look at philosophical skepticism the same way. It is illusion masquerading as metaphysical truth.

    ReplyDelete
  42. dguller,

    Of course, I share your belief in the futility of philosophical skepticism. And you give a good reason for refusing to believe it: Otherwise, there would be rank confusion among men. What you have provided is a short, sweet, and decisive philosophical argument against skepticism. Science itself cannot provide this argument; as Dr. Feser says, it assumes it from the get-go.

    It follows that there is, and must be, knowledge other than scientific knowledge - because science itself depends on it as its foundation (the distinction between appearance and reality that you explore in the second paragraph, for example, is a philosophical distinction).

    Scientism denies this extra-scientific basis of knowledge. It claims that only scientific knowledge qualifies as true knowledge; in this it is not only wrong, but ultimately destructive of science itself.

    ReplyDelete
  43. "What you have provided is a short, sweet, and decisive philosophical argument against skepticism."

    No, what he has provided is a simplistic, unsophisticated argument against skepticism--not that I necessary fault him for that, in a comments box. Dguller, you underestimate the power of conformity and consensus. You don't give the pattern-making properties of the brain and senses enough consideration, either. Finally, I would suggest that you try engaging with Nietzsche, some time.

    At any rate, as one who finds Nietzschean perspectivism "decisive", I am sure I am in the wrong place. I found this blog while searching for critiques of scientism. Because my skepticism extends to all human forms of knowledge, including science (or scientism), I look forward to reading Edward Feser's take on the subject, regardless of how much I might disagree with him or others here on (most?) other matters..

    ReplyDelete
  44. Dmt117:

    >> Of course, I share your belief in the futility of philosophical skepticism. And you give a good reason for refusing to believe it: Otherwise, there would be rank confusion among men. What you have provided is a short, sweet, and decisive philosophical argument against skepticism. Science itself cannot provide this argument; as Dr. Feser says, it assumes it from the get-go.

    I disagree. I experience an external world that operates according to regular patterns and behavior. I do not just assume it, but experience it all the time. That is my default and primary state, and it is yours, too. There are some times when this default state can waver, especially when exposed to the pathogen of philosophical skepticism, but once I have immunized myself against it by realizing that it is just an illusion masquerading as knowledge, I am back to the primary state of directly experiencing the world. This is not a philosophical argument – although I did provide those, too – but an empirical fact, which is not weakened in any way by fictional accounts and fantastical tales.

    >> It follows that there is, and must be, knowledge other than scientific knowledge - because science itself depends on it as its foundation (the distinction between appearance and reality that you explore in the second paragraph, for example, is a philosophical distinction).

    First, there is knowledge other than scientific knowledge. Mathematics and logic are good examples. My position is actually that the best methodology we have to understand the natural world is the scientific enterprise. However, with regards to mathematics and logic, the only way to know if they apply to the world is to engage in scientific inquiry to determine whether our observations of the world match the predictions of mathematics and logic.

    Second, the appearance and reality distinction is an everyday distinction that does not require philosophy. Any time I see something that isn’t there, the distinction is operative. It is operative in children by four years of age as observed in false-belief tasks. I highly doubt that these children are engaging in philosophy. Of course, philosophy has taken this everyday idea and extended it into fantastical ontological realms, but its root is in our common sense and naïve experience of the world.

    >> Scientism denies this extra-scientific basis of knowledge. It claims that only scientific knowledge qualifies as true knowledge; in this it is not only wrong, but ultimately destructive of science itself.

    Scientism, as far as I understand it, says that scientific knowledge is the best knowledge we have about the universe and how it works. The additional claim is that the universe is all that there is, and thus scientific knowledge is the best knowledge of all that actually exists. Beyond that, there is sheer speculation, because we cannot confirm or falsify by any means, other than logical coherence or incoherence, whether the claims are true or false. This is problematic, because coherence is necessary, but not sufficient, for truth, and thus we are stuck in limbo.

    ReplyDelete
  45. Kevin:

    >> No, what he has provided is a simplistic, unsophisticated argument against skepticism--not that I necessary fault him for that, in a comments box.

    Care to elaborate about the faults in my argument?

    >> Dguller, you underestimate the power of conformity and consensus. You don't give the pattern-making properties of the brain and senses enough consideration, either. Finally, I would suggest that you try engaging with Nietzsche, some time.

    Actually, I do not “underestimate the power of conformity and consensus” and “the pattern-making properties of the brain and senses”. Believe me. I am putting together a grand rounds presentation in the hospital where I work that emphasizes these very points!

    That being said, scientific inquiry is the only form of inquiry that acknowledges from the onset all the factors that you mentioned, as well as a number of other confounding factors – psychological and otherwise – that could distort a scientific conclusion. That is why it takes steps to control for these factors to minimize their influence over the effects observed. It also acknowledges the role of chance and probability, which is why studies typically have to be replicated, because there is always a chance that the positive or negative finding is just luck. And in no other discipline is there so much credit given to those who debunk well-established beliefs. In fact, a number of prestigious awards and honors to go those who successfully do so!

    >> At any rate, as one who finds Nietzschean perspectivism "decisive", I am sure I am in the wrong place.

    I have read some Nietzsche, and the parts that I found persuasive were trivially true, and the parts that I found unpersuasive were obviously false. And as to perspectivism, what is the big deal? We experience the world from a particular perspective. So what? What catastrophic consequences follow from this? Do we have to observe the world from a View From Nowhere to have any knowledge of it?

    ReplyDelete
  46. Dguller:

    No time to engage in depth with you on this; if you wish to take that as a concession or a weakness, then so be it. I'll just say the following, (semi) briefly:

    I alluded already to the weaknesses in your argument: You seem to assume that the regularity of your perceptual apparatus and senses means that there is one true world out there, and that you happen to observe it correctly, one way or another. Your argument also relies upon consensus ("How would communication be possible, if we didn't all see the same things accurately?").

    Further, you have already conditioned yourself to reject out of hand any anomalies you may experience, by dismissing them as "illusions". Your position rests upon faith and dogma, ultimately. Nothing necessarily wrong with that, of course, but you should at least acknowledge the fact that you are making many acts of faith and metaphysical presuppositions.

    As for Nietzsche, if you do not see why perspectivism challenges your views, then you need to read more than the "some" you have, to date. Best of luck in your exploration, should you choose to undertake it.

    ReplyDelete
  47. Kevin:

    >> I alluded already to the weaknesses in your argument: You seem to assume that the regularity of your perceptual apparatus and senses means that there is one true world out there, and that you happen to observe it correctly, one way or another. Your argument also relies upon consensus ("How would communication be possible, if we didn't all see the same things accurately?").

    I do not assume anything. I experience the regularity of an external world. Assumption is only relevant if all I can experience are disconnected bits of sensory experience that are divorced from the world. That is not the default state, but the result of fallacious philosophical arguments. Remember: we do not begin as philosophers, but as human beings interacting with the world. It is only after we are exposed to philosophical skeptical fantasies that our initial confidence in our perception of the world becomes momentarily shaken. However, I see no need to question the veracity of my perception of an external world on the basis of fiction.

    >> Further, you have already conditioned yourself to reject out of hand any anomalies you may experience, by dismissing them as "illusions". Your position rests upon faith and dogma, ultimately. Nothing necessarily wrong with that, of course, but you should at least acknowledge the fact that you are making many acts of faith and metaphysical presuppositions.

    No, they are illusions, because they are experiences of things that do not exist. For example, when I see a bent stick in the water, I know that the stick is not, in fact, bent, because when I pull it out of the water, it is straight again. The water distorted my visual experience of the stick into an illusory experience. I mean, this is just common sense. It is not an anomaly that requires the violation of everything that I know, even if a philosophical fairy tale can be imagined to explain an alternative explanation for it. I can also imagine invisible unicorns distorting the photons, but would you really take this hypothesis seriously? I doubt it.

    >> As for Nietzsche, if you do not see why perspectivism challenges your views, then you need to read more than the "some" you have, to date. Best of luck in your exploration, should you choose to undertake it.

    Thanks, and good luck to you, as well. Thanks for sharing your thoughts.

    ReplyDelete
  48. Hi

    I'm in the process of reading TLS and am very much enjoy it. I came across the video linked below on http://aquinasonline.com/ where the author, claiming to be an authority on Thomism, posits that the first way is refuted by modern physics since all interactions are accounted for via the universe being a closed system where change is simply a transfer of energy within the closed system.

    I'm hoping someone with more Thomistic experience can provide some insight into the video's claim.

    Thanks,
    Tom


    http://www.youtube.com/watch?v=urj4K1blY3w

    ReplyDelete
  49. dguller,

    I think our differences are mostly semantic at this point. I have a much broader definition of philosophy than do you; I include the "everyday distinctions" that we all know as philosophical knowledge. In fact, they are the foundation of philosophical knowledge. Whether we call them philosophical or not, I think we agree that science can only get going in terms of them. It is just this that some folks deny; they claim that these everyday distinctions are not knowledge because they are not conclusions of science.

    ReplyDelete
  50. Kevin,

    Just wondering what it is about your argument that makes it rise above faith and dogma, when everyone else's fails to do so. This is an honest question...

    ReplyDelete
  51. dguller,

    Here's an exercise for you that (I hope) illustrates a philosophical point:

    Lets suppose there's a car accident with 30 eyewitnesses.

    Each witness has his own version of what happened.

    Some vary widely.

    Is there some standard by which all versions can be judged?

    Does that standard exist in and of itself or is it a construction of minds?

    Does that standard exist even after the accident is over?

    What would you call that standard?

    Can all 30 witnesses be wrong even if they all agree?

    I look forward to your answers.

    ReplyDelete
  52. Dmt117:

    >> I think our differences are mostly semantic at this point. I have a much broader definition of philosophy than do you; I include the "everyday distinctions" that we all know as philosophical knowledge. In fact, they are the foundation of philosophical knowledge. Whether we call them philosophical or not, I think we agree that science can only get going in terms of them. It is just this that some folks deny; they claim that these everyday distinctions are not knowledge because they are not conclusions of science.

    Fair enough, except that I would not say that these “everyday distinctions” are unscientific. They are grounded in our ordinary activity of inquiry, of which scientific inquiry is a particularly sophisticated and rigorous example. So, the two are not mutually exclusive, but are part of the same activity of empirical engagement with the world and observing it to test ideas and hypotheses, which is something that we all do, whether scientist or not. My objections were based upon your apparent claim that these “everyday distinctions” and activities somehow stand in opposition or contradiction to science when that is not the case at all. They are all part of the same continuum of human inquiry and engagement with the world.

    ReplyDelete
  53. Daniel:

    >> Is there some standard by which all versions can be judged?



    Ultimately, the truth of the respective versions would depend on whether they correspond to the state of affairs that actually occurred in the accident.

    If I was looking into the 30 witnesses, then I would group them according to which were closer to the accident, which were facing the accident, which were directly paying attention to the accident, which lacked any sensory impairment, which lacked any cognitive impairment, which lacked any incentive to lie about the accident, and so on. So, the consensus of the most reliable witnesses would be the starting point for this inquiry, and then I would look for corroboration for their story with forensic evidence, video evidence, and so on. The idea is that this process has the best chance of getting to the truth about the matter.

    I suppose I would say that the standard would be the construction of events that best captures the majority opinion of the most reliable witnesses, and that is consistent with other evidence from the scene.

    >> Does that standard exist in and of itself or is it a construction of minds?



    The standard does not exist in and of itself. If there were no human beings, then there would be no human inquiry, which means that there would be no standards of evidence, for example. I mean, does basketball exist in and of itself, or is it an activity that depends upon human beings? Human inquiry is an activity that we perform when we want to get to the truth of something. It depends upon us and would not occur without us.

    >> Does that standard exist even after the accident is over?



    Sure, as long as there are still human beings seeking to understand their world.

    >> What would you call that standard?



    Rules of human inquiry, I suppose.

    >> Can all 30 witnesses be wrong even if they all agree?

    It’s possible, but highly unlikely, especially with regards to the major details of the event. Certainly, there may be confusion over minor details, such as the color the clothing of the drivers, and so on, but that is of little consequence. However, I can imagine some scenarios where what you describe could play out. One would be that the person who caused the accident was the son of a dictator who would punish the witnesses for implicating his son, and so the witnesses agree to lie about what they saw. Another would be that they hate the victim in the accident, and subtly influence each other to lie in such a way to implicate him as the cause, perhaps to the point that they are unaware of their self-deception. So, it is possible, but barring unusual circumstances, I would trust the witnesses, especially if they all agreed on the big details, and especially on the small details.

    ReplyDelete
  54. Off topic, but some her may appreciate the story of how Thomas Kuhn threw an ashtray at a Grad student.

    ReplyDelete
  55. When the subject that science itself is sufficient and needs no philosophical grounding comes up, the argument usually offered is that science works. This amounts to asserting induction is true by using an inductive argument. This is obviously circular, but for those who don't see that, one could argue for anti-induction, that when one fails to observe a regularity in nature, that establishes its truth. And as proof one could offer an anti-inductive argument. Since anti-induction has not been observed to work in the past, it must therefore be true,

    ReplyDelete
  56. Fair enough, except that I would not say that these “everyday distinctions” are unscientific. They are grounded in our ordinary activity of inquiry, of which scientific inquiry is a particularly sophisticated and rigorous example.

    When I watch TV, I am engaging in what could be called an empirical activity. It's not "scientific inquiry", even a particularly lazy and fun form of it.

    ReplyDelete
  57. Crude:

    >> When I watch TV, I am engaging in what could be called an empirical activity. It's not "scientific inquiry", even a particularly lazy and fun form of it.

    If you are watching TV to gather information about the world, then yes, you are engaged in inquiry. TV can be a source of data that can contribute to understanding. But as you mentioned, it is not a particularly good way to go about it, especially if that is your sole source of information. However, if you are watching TV to be entertained, then that is not inquiry of any kind. As you know, we do a number of activities, only some of which are geared towards learning the truth about something.

    ReplyDelete
  58. If you are watching TV to gather information about the world, then yes, you are engaged in inquiry.

    Watching CNN is not "scientific inquiry". Merely having information related to me is not "scientific inquiry", even if I can quote it later.

    Likewise, watching TV does result in "knowledge", even if I'm not watching it "as a form of inquiry". Yes?

    ReplyDelete
  59. "Further, you have already conditioned yourself to reject out of hand any anomalies you may experience, by dismissing them as "illusions"."

    "No, they are illusions, because they are experiences of things that do not exist. For example, when I see a bent stick in the water, I know that the stick is not, in fact, bent, because when I pull it out of the water, it is straight again. The water distorted my visual experience of the stick into an illusory experience. I mean, this is just common sense."

    The stick in the water does not exist as bent only in relation to how you perceive it when you take it out. But how do you know that the illusion isn't the way you experience the stick when it's out of the water and not in it? In fact, how have you come to think that what is contrary to your (or every day) 'regular expirience' is what is illusory? It could be the case that the stick is really bent, but when it's out of the water, something causes (and distorts) it to appear as straight.

    The judgement you pass on this question will be done philosophically not empirically.

    "My objections were based upon your apparent claim that these “everyday distinctions” and activities somehow stand in opposition or contradiction to science"

    They don't stand in opposition to science (as Aristotle conceived of it)), but to scientism (i.e. only that which is empirically veriable is knowledge proper)

    ReplyDelete
  60. empirically verifiable**

    ReplyDelete
  61. Crude:

    >> Watching CNN is not "scientific inquiry". Merely having information related to me is not "scientific inquiry", even if I can quote it later.

    I never said it was scientific inquiry. I said it was inquiry, but only if you have a specific question in mind that you require information to decide, and if you believe that that information is available on TV. So, you are correct that inquiry is not a passive process, but rather an active engagement with the world.

    >> Likewise, watching TV does result in "knowledge", even if I'm not watching it "as a form of inquiry". Yes?

    Yup. We can passively acquire information from our environment, and that information can be a form of knowledge, especially if it is verifiable by other sources. Without that additional verification, the information is merely anecdotal, and thus can be affected by multiple biases and distortions. Inquiry, on the other hand, is the active engagement with the world to acquire evidence to confirm or reject a hypothesis or belief.

    ReplyDelete
  62. I never said it was scientific inquiry.

    You said: They are grounded in our ordinary activity of inquiry, of which scientific inquiry is a particularly sophisticated and rigorous example.

    That seems to strongly imply that our everyday inquiry is scientific inquiry. So, glad to clear that up.

    We can passively acquire information from our environment, and that information can be a form of knowledge, especially if it is verifiable by other sources.

    It's knowledge, but it's "especially" knowledge if it's verifiable? What if I don't verify it?

    It seems to me that once you admit that everyday experience grants knowledge, and that said knowledge is not the result of scientific inquiry, then the amount of knowledge from scientific inquiry is utterly swamped by the knowledge from non-scientific inquiry.

    ReplyDelete
  63. Crude:

    >> That seems to strongly imply that our everyday inquiry is scientific inquiry. So, glad to clear that up.

    Everyday inquiry is primary, because we all engage in it in our daily lives. Scientific inquiry is a more sophisticated and rigorous form of inquiry, which is partly why it has been so successful.

    >> It's knowledge, but it's "especially" knowledge if it's verifiable? What if I don't verify it?

    It is knowledge in the sense that it is something that you know, but it is not a justified true belief, which is the more appropriate type of knowledge that we should be aiming for, Gettier examples aside. And if you cannot verify a piece of information, then I do not think that it can count as knowledge. I mean, I can hear someone on the street say anything, but unless I can corroborate it somehow, such as by looking it up in a book, consulting experts, or whatever, then it is just information and not knowledge.

    >> It seems to me that once you admit that everyday experience grants knowledge, and that said knowledge is not the result of scientific inquiry, then the amount of knowledge from scientific inquiry is utterly swamped by the knowledge from non-scientific inquiry.

    I actually do not know the figures involved, but what you say seems right. Remember, all I am saying is that scientific inquiry is the best method we have to discover truths about the empirical world. It does not follow that it is always right, or that it is always used, or even necessary to answer every question that we have. I mean, if I want to know where my wife is, then I will call her on her cell-phone. I do not need to engage in a rigorous scientific study to answer that question.

    ReplyDelete
  64. Everyday inquiry is primary, because we all engage in it in our daily lives. Scientific inquiry is a more sophisticated and rigorous form of inquiry, which is partly why it has been so successful.

    And everyday inquiry is not scientific inquiry. Yep. What isn't a "form of inquiry" in some sense anyway? Even raw subjective experience seems to be giving me information of a sort about something.

    It is knowledge in the sense that it is something that you know, but it is not a justified true belief, which is the more appropriate type of knowledge that we should be aiming for, Gettier examples aside. And if you cannot verify a piece of information, then I do not think that it can count as knowledge.

    I think I had a pretty good pizza last week. Dr. Oetker brand, mushroom topping. How do I verify this?

    Pretty sure FDR was a president of the US at one point. How do I verify that?

    Pretty sure our sun originally wasn't there, and came into being at some time in the past. How do I verify that?

    I'm not going anywhere with this. I'm just curious how to get this verification thing going on.

    ReplyDelete
  65. Crude:

    >> And everyday inquiry is not scientific inquiry. Yep. What isn't a "form of inquiry" in some sense anyway? Even raw subjective experience seems to be giving me information of a sort about something.

    Inquiry is not passive. It is having a question in mind, and actively seeking evidence to answer the question. And inquiry is not the only way to acquire knowledge, but it is better than passive experience, because one can consciously consider ways that our experience may be wrong, and possibly correct for those confounding factors. The more rigorous and sophisticated these methods become, the more one enters into scientific inquiry, which is just an extension of everyday inquiry. They are not different, but fundamentally the same. Just as a Chevrolet and a Mercedes are both cars, the latter is of superior quality.

    >> I think I had a pretty good pizza last week. Dr. Oetker brand, mushroom topping. How do I verify this?

    You can search your memory for any recollection of said event. You can call the pizza place to see if they have a record of the order. You can speak to the delivery man to see if he remembers you. You can try to find your receipt. You can call anyone you may have commented to about your pizza. You can speak to friends who visited you and may have seen the pizza there. You can search for empty pizza boxes in your garbage.

    >> Pretty sure FDR was a president of the US at one point. How do I verify that?

    You can read a history book. You can look at video footage and photographs of FDR. You can call the White House to see if they will confirm his presidency. You can read texts that claim that FDR was never president, and compare their evidence to the evidence for the claim that he was president, and assess which has the stronger case.

    >> Pretty sure our sun originally wasn't there, and came into being at some time in the past. How do I verify that?

    I’m not too sure. Ask an astrophysicist. I think he or she will have a better answer than me.

    >> I'm not going anywhere with this. I'm just curious how to get this verification thing going on.

    Hope this helps.

    ReplyDelete
  66. Inquiry is not passive.

    Sure, but am I 'inquiring' when I just watch TV? Because I sure seem to be getting knowledge/information. If so, it looks like actively inquiring isn't necessary to get information/knowledge.

    You can search your memory for any recollection of said event. You can [...]

    Yeah. Which one of those things verifies "I had that Dr. Oetker pizza last week and it was pretty good."? I already know my recollection, but obviously that can be mistaken. And the rest of the examples, for both the pizza and FDR, seem - at best - as if they can fit in with the claim, and that makes some ideal assumptions (What if the pizza guy says "no"? Maybe he's forgetful). But what's verifying it rather than merely acting as partial corroboration?

    Or is your version of "verification" one where it's still entirely possible to be wrong?

    I’m not too sure. Ask an astrophysicist. I think he or she will have a better answer than me.

    Alright, this will be helpful: Do you, right now and prior to such question-asking and research, have knowledge that the sun came into existence at some point? You said that if it's not verifiable, it's not knowledge. Well, is it not knowledge until you verify it? (But just 'being verifiable in principle' seems too weak to be doing work here.)

    What if the astrophysicist says "Well, the best we can do is assume that laws hold at all times, and then based on this information we can extrapolate backwards and, if we do our math right..." Is that a verification?

    ReplyDelete
  67. Crude:

    >> Sure, but am I 'inquiring' when I just watch TV? Because I sure seem to be getting knowledge/information. If so, it looks like actively inquiring isn't necessary to get information/knowledge.

    You are correct. But, actively inquiring is a better way to get knowledge than passively absorbing information by osmosis, because you have no way of evaluating the source of the information when in a passive state. Evaluation is an active process, after all.

    >> Or is your version of "verification" one where it's still entirely possible to be wrong?

    Of course you can be wrong. Verification is not an all-or-nothing process, which I mentioned on the previous thread at March 5, 2011 7:25 AM. Here’s what I wrote:

    “Verification is not an all-or-nothing phenomenon, but is a matter of likelihood, given the evidence. 



    “Say I want to verify whether some state of affairs X is actually occurring in the world. I will direct my perspective towards X to see if X can be experienced directly by my senses or indirectly by technological extensions of my senses (e.g. microscopes, telescopes, etc.). If X can be experienced in this way, then its likelihood of being real goes up. 



    “Now, if there are confounding factors that could be distorting my perception, whether these are cognitive biases or distortions, hallucinations, or any other quirks of our psychology, then these will have to be ruled out, if possible. If these factors are present, then the likelihood of X being real goes down. X must also be compared to our background experiences and conceptual framework to see if it contradicts it in any important way.



    “The likelihood never reaches 100%, because it is always possible that we could be wrong, but it is about doing the best we can, which will never be perfect. So, empirical verification is about looking for empirical evidence supporting X and ruling out empirical confounding factors that could make X an illusory phenomenon.



    “I think this is important to clarify, because you seem to assume that if I empirically verify X, then it is verified for all time and can never be revised. Maybe I am wrong about your assumptions, but just in case, I wanted to clarify my understanding of this matter.”

    Let me know what you think.

    ReplyDelete
  68. Crude:


    >> Alright, this will be helpful: Do you, right now and prior to such question-asking and research, have knowledge that the sun came into existence at some point?

    Yes, I do.

    >> You said that if it's not verifiable, it's not knowledge. Well, is it not knowledge until you verify it? (But just 'being verifiable in principle' seems too weak to be doing work here.)

    It is not knowledge until it is verified. The less verification, the more speculation.

    >> What if the astrophysicist says "Well, the best we can do is assume that laws hold at all times, and then based on this information we can extrapolate backwards and, if we do our math right..." Is that a verification?

    Sure. Especially if there are predictable consequences to this theory that are subsequently observed to be true, such as observing other stars being formed in a way that is consistent with this theory with information acquired from telescopes, for example. I would count that as verification.

    ReplyDelete
  69. dguller,

    You claim said: "If you cannot verify a piece of information, then I do not think that it can count as knowledge."

    Can you verify that proposition?

    ReplyDelete
  70. Martin asked,

    "dguller,

    You claim said: "If you cannot verify a piece of information, then I do not think that it can count as knowledge."

    Can you verify that proposition?"

    I don't know what dguller means by "verify". If he means "have a sense experience of", then he can't verify that proposition. However, by "verify" he might mean "show to be true via sense experience, scientific testing, or induction". If he means that, then he could verify that proposition via induction.

    He might also mean something simpler by "verify" such as "show to be true". In that case, the proposition itself might not exactly be verified, but perhaps it can be shown to be a proposition that most of us are logically committed to, given the kinds of things we count as instances of knowledge in our day-to-day life.

    ReplyDelete
  71. dguller,

    "Further, you have already conditioned yourself to reject out of hand any anomalies you may experience, by dismissing them as "illusions"."

    "No, they are illusions, because they are experiences of things that do not exist. For example, when I see a bent stick in the water, I know that the stick is not, in fact, bent, because when I pull it out of the water, it is straight again. The water distorted my visual experience of the stick into an illusory experience. I mean, this is just common sense."

    The stick in the water does not exist as bent only in relation to how you perceive it when you take it out. But how do you know that the illusion isn't the way you experience the stick when it's out of the water and not in it? In fact, how have you come to think that what is contrary to your (or every day) 'regular expirience' is what is illusory? It could be the case that the stick is really bent, but when it's out of the water, something causes (and distorts) it to appear as straight.

    The judgement you pass on this question will be done philosophically not empirically.

    "My objections were based upon your apparent claim that these “everyday distinctions” and activities somehow stand in opposition or contradiction to science"

    They don't stand in opposition to science (as Aristotle conceived of it)), but to scientism (i.e. only that which is empirically veriable is knowledge proper)

    ReplyDelete
  72. Martin:

    >> Can you verify that proposition?

    One way to verify this would be to have two groups of people, matched for demographics, intelligence, education, and so on. Then I would instruct the first group to prioritize verifying information that is provided to them to ensure that it is true (i.e. by looking it up in a text, consulting an expert, actually going somewhere to confirm it themselves, etc.) and the second group to naively believe anything that is told to them as true. One could then analyze how many beliefs that were acquired since the start of the experiment were true in the two groups and see if the former had more true beliefs than the latter, which I suspect would likely be the case.

    ReplyDelete
  73. dguller,

    Ah, so verification simply means having justification, not necessarily empirical verification.

    ReplyDelete
  74. Martin:

    >> Ah, so verification simply means having justification, not necessarily empirical verification.

    It depends on the claim being justified. If the claim is about the empirical world, then the justification must be empirical verification. If the claim is about a mathematical proof, then the justification will be mathematical verification.

    ReplyDelete
  75. So how do you verify the following propositions?

    "The external world is real."

    "The sun will rise tomorrow."

    "People other than myself are conscious."

    These are claims about the empirical world, but I don't see how you can verify them empirically.

    ReplyDelete
  76. >One way to verify this would be to have two groups of people, matched for demographics, intelligence, education, and so on. Then I would instruct the first group to prioritize verifying information that is provided to them to ensure that it is true (i.e. by looking it up in a text, consulting an expert, actually going somewhere to confirm it themselves, etc.) and the second group to naively believe anything that is told to them as true. One could then analyze how many beliefs that were acquired since the start of the experiment were true in the two groups and see if the former had more true beliefs than the latter, which I suspect would likely be the case.

    NONE of the above is a verification of the following proposition:

    "If you cannot verify a piece of information, then I do not think that it can count as knowledge."

    Just. Stop. Please.

    ReplyDelete
  77. Martin:

    >> "The external world is real."

    I experience an external world.

    >> "The sun will rise tomorrow."

    I cannot verify a future event until it happens. I can make a prediction based on past experience, though, but that is not the same as verifying it. I think that verification only applies to past or present events.

    >> "People other than myself are conscious."

    I experience them as conscious. It turns out that this is partly due to the activation of mirror neurons that activate the neurobiological pathways that we use to generate our own internal experience. This is also how empathy and compassion are possible. In other words, we directly experience within ourselves what others feel, but from our own perspective.

    One of the ways that we know this is by studying people who are unable to identify other human beings as having conscious mental states, such as those with autism. Not only can this neurological pathway be inhibited, such as in autism, but it can also be activated at the wrong time. For example, when we beg and plead with our cars when they do not start, we are activating this mentalizing capacity in an inappropriate setting, because cars do not share our neurobiological underpinning, and thus cannot be consciously aware of our pleading at all.

    This can further be confirmed by the fact that we all share a language in which we describe our internal states as conscious, and there is not rank confusion between us when using this language.

    So, we experience them as conscious, we share a common brain-body anatomy, and we share a language that uses “conscious” without utter confusion. These are all empirical observations, I think.

    ReplyDelete
  78. Brian:

    >> NONE of the above is a verification of the following proposition: "If you cannot verify a piece of information, then I do not think that it can count as knowledge."

    Why not? We have experience of holding beliefs that are justified and verified, and we have experience of holding beliefs that are unjustified and unverified. Which are more likely to be true? I proposed an empirical study to show whether naively believing anything and everything that one is told versus actively inquiring into whether what is told is justified or not. If it turns out that actively engaging in verification and justification of one’s beliefs results in more true beliefs than blind following, then one should verify and justify one’s beliefs, if one wants to maximize one’s chances of having true beliefs.

    Again, this is not an all-or-nothing thing. There is no 100% certainty or guarantees in our search for truth. The best that we can do is to maximize our chances of success by adhering to standards of justification that have been demonstrated to increase our chances of having true beliefs about the world.

    ReplyDelete
  79. "I experience an external world."

    It's nice how far Matrix technology has come, hasn't it? What you are experiencing is the sensation of firing rods and cones in the back of your eyes, which are being triggered by (possibly) photons, which are coming from... the external world?

    Perhaps.

    Could also be hooked up to a machine somewhere. You cannot empirically verify that the external world really exists, independent of yourself. Yet I'm sure you hold that it does. Isn't this inconsistent?

    "I experience them as conscious."

    You cannot possibly experience people as concious, unless you become one of them. You know that they make sounds, and they react AS IF they were in pain, pleasure, etc. But you have no way of empirically verifying that they are actually experiencing consciousness.

    ReplyDelete
  80. Martin:

    >> It's nice how far Matrix technology has come, hasn't it? What you are experiencing is the sensation of firing rods and cones in the back of your eyes, which are being triggered by (possibly) photons, which are coming from... the external world?

    You got it … except for the Matrix technology part. That was actually, you know, fiction. The fact is that you and I both experience the world. You can pretend that your philosophical fantasies somehow change that fact, but when you are living your life, you do not pretend that there is an external world.

    >> Could also be hooked up to a machine somewhere. You cannot empirically verify that the external world really exists, independent of yourself. Yet I'm sure you hold that it does. Isn't this inconsistent?

    Hook someone up to a machine first, and we can see what they experience. That would be an interesting experiment to conduct, but I doubt that it would ever occur due to ethical concerns. Until then, it is yet another skeptical fairy tale. I am not impressed by “what-if” possibilities. There are an infinite number of them, and the onus is not upon me to refute all of them before I can rely upon my ordinary perception of the world. You have to show that your particular fantasy actually applies to the world before I have to take it seriously. Until then, it is science fiction. Maybe you are kept up at night by fiction, but I am only interested in facts when trying to navigate within the world to achieve my goals.

    >> You cannot possibly experience people as concious, unless you become one of them. You know that they make sounds, and they react AS IF they were in pain, pleasure, etc.

    No, I do experience them as conscious. Now, that does not mean that I have to experience THEIR consciousness from their perspective. That is impossible, but fortunately is also unnecessary. I know that I experience them as conscious, and I know that there is an explanation for why I do so. I am hardwired to be in sync with them by virtue of our shared neurobiological structure, which incredibly can attain dyadic interactions through feedback loops between us, which are mediated by our bodies and brains. Social neuroscience is a fascinating subject, and makes real contributions to our knowledge of how we experience and interact with one another.

    And I do not experience their external bodily movements and sounds, and then pretend that they are in pain. When another person is in pain, I also wince in pain. Why? Because my mirror neurons have activated by own pain experience via the insula, which is one of the roots of human empathy and compassion. In other words, my experience of another’s mental state is not intellectual or conceptual, but VISCERAL. We are hardwired to be attuned to one another, and are capable of interacting in a coherent fashion by virtue of experiencing both ourselves and others as having conscious minds. As I mentioned earlier, you can see this clearly in those who suffer from brain dysfunction secondary to injury, infection or congenital defect.

    >> But you have no way of empirically verifying that they are actually experiencing consciousness.

    Sure I can. If I experience them as conscious, and they have the same underlying biological structure as me, and they behave in a way that I can predict and understand as secondary to mental states, and they describe themselves as having consciousness, then that is enough empirical confirmation for me to conclude that they are, in fact, conscious. I do not have to experience their consciousness from their first-person perspective at all. Could I be wrong? Sure. But the possibility that I am wrong is not sufficient to show that I am, in fact, wrong, especially when there is an abundance of evidence to confirm my view.

    ReplyDelete
  81. A sufficiently advanced AI will also appear conscious, even though it may be totally different in neural structure to a human being and may just be running very complicated algorithms. Gut instinct, pure assumption and human empathy are not enough to prove another being is conscious.

    People often believe inanimate objects are conscious. A while ago a young lonely gentleman in Japan married his favourite female dating simulation game character. He believes she really loves him. Of course this is an obvious delusion but it is a tendency to give human or our personal characteristics to other people, inanimate objects and animals which could be part of this whole belief in others' consciousness or self-awareness. At the end of the day, whether hard wiring is true or not, it is still perceived through the lens of our own minds and senses and we just have to make that assumption that what we experience and what our instruments tell us is real. It's a matter of faith, a pretty transparent one but faith anyhow.

    ReplyDelete
  82. Anonymous:

    >> A sufficiently advanced AI will also appear conscious, even though it may be totally different in neural structure to a human being and may just be running very complicated algorithms. Gut instinct, pure assumption and human empathy are not enough to prove another being is conscious.

    You are assuming that consciousness must be of a human variety. Even if an advanced AI does achieve conscious awareness, then it does not follow that it will match ours. And what does this matter? We are talking about human consciousness, right? How do I know whether another human being is conscious? For all the reasons that I said. You have given me no reason to think that those reasons are not sufficient. Whether those reasons also apply to computers or aliens, or whatever, is an open question that cannot be answered at this time.

    >> People often believe inanimate objects are conscious. A while ago a young lonely gentleman in Japan married his favourite female dating simulation game character. He believes she really loves him. Of course this is an obvious delusion but it is a tendency to give human or our personal characteristics to other people, inanimate objects and animals which could be part of this whole belief in others' consciousness or self-awareness.

    I made this very point in my comments above. Our mentalizing capacity can misfire and perceive conscious awareness in something that lacks consciousness. I used the example of arguing with a stalled car. It does not imply that our mentalizing capacity also misfires when it applies to human beings. I would argue that it does its job perfectly well in that case, and that the misfires are parasitic upon the normal functioning. Similarly, hallucinations are parasitic upon the mechanisms of normal perception being activated in inappropriate circumstances. That does not imply that we are always hallucinating, though.

    >> At the end of the day, whether hard wiring is true or not, it is still perceived through the lens of our own minds and senses and we just have to make that assumption that what we experience and what our instruments tell us is real. It's a matter of faith, a pretty transparent one but faith anyhow.

    It is not. It is a matter of empirical experience, much like the existence of an external world. However, this is impossible if you start with nothing but your subjective experience, and then try to connect it to some external reality. Fortunately, this is nothing but a philosophical fairy tale, and should be ignored. The reality is that we are embedded in the world, and that our cognitive and perceptual capacities interact with that world in a largely reliable fashion, which is why they can be trusted for the most part.

    ReplyDelete
  83. dguller: "Ultimately, the truth of the respective versions would depend on whether they correspond to the state of affairs that actually occurred in the accident."

    That is correct. The "truth" is "what actually occurred".

    "The standard does not exist in and of itself. If there were no human beings, then there would be no human inquiry, which means that there would be no standards of evidence"

    So the truth about what happened would not exist unless there are humans around? That's bizarre. The truth about what happened is just exactly that: "what actually happened". It exists whether we are around or not.

    "Rules of human inquiry, I suppose."

    So the standard against which all accounts are judged, (which is "what actually happened") has now somehow morphed into "rules of human inquiry"?

    How does that work?

    The point of this was to illustrate that there is an ultimate reality ("what actually happens/happened") and there are multiple ways of finding it out. Direct empirical observation (the 30 witnesses) can get you part of the way there, (so long as there are eyewitnesses) but even 30 eyewitnesses will not know every detail (some of which may be very important.) The witnesses could not tell you, for example, that the steering column broke on one vehicle and that that was the reason one driver veered into the other.

    So when direct empirical observation has given all it has to give, other forms of inquiry must take over. The scene of the accident is examined, the vehicles are poured over, the bodies of the drivers are examined for injuries. Reason, deduction, logic - these then spring into play. From the evidence - the hard evidence as it were - a scenario is developed. Reasonable deductions are made. These "cold, hard facts" often trump any eyewitness accounts and become a kind of test against which the direct observations are judged. This is because humans are notoriously unreliable (due to the very filters we've discussed at length in this and other threads.)

    So even though it is based on empirical evidence, the deduction process itself is NOT direct observation. This is key. There are other methods of arriving at the truth beside direct observation. There is reasonable deduction based on empirically observable facts and established laws of physics, geometry, mechanical engineering, logic, etc.

    Which leads me to Thomas Aquinas (please bear with me here.)

    Aquinas gave us five proofs of God's existence. These proofs were based on established, observable principles of nature - causation; motion and change; contingency; degrees of perfection; and the repeatable, reliable, tendencies of nature. From these things he made reasonable deductions: A chain of causation must terminate in an uncaused cause; A chain of motion must terminate in an unmoved mover; Intentionality requires a mind; and etc.

    These are not "pie in the sky", "brain in a vat", "pretend the real world doesn't exist" philosophical arguments of the kind you often eschew (though those have their place.) No these are real world, empirically based, reasoned deductions that have their roots in fundamental, simple principles that are undeniable.

    These, my friend, are akin to the reconstruction of the accident from its remnants - and just as scientific - unless, of course, you exclude such deductions from science. In which case, you'd tie the hands of every forensic team, every archaeologist, every cosmologist - anyone who makes deductions based on existing evidence in order to decipher the truth about what actually happened some time in history (or what's actually happening right now.)

    So... If you really want to get to the truth, don't exclude reasoned metaphysical deductions like Aquinas'. You may just miss it if you do!

    ReplyDelete
  84. Daniel:

    >> So when direct empirical observation has given all it has to give, other forms of inquiry must take over. The scene of the accident is examined, the vehicles are poured over, the bodies of the drivers are examined for injuries.

    Well, these are all empirical observations. They are not another form of inquiry. They are gathering different types of evidence, some eyewitness testimony, and some forensic evidence. It is all part of the same process of inquiry.

    >> Reason, deduction, logic - these then spring into play. From the evidence - the hard evidence as it were - a scenario is developed. Reasonable deductions are made. These "cold, hard facts" often trump any eyewitness accounts and become a kind of test against which the direct observations are judged. This is because humans are notoriously unreliable (due to the very filters we've discussed at length in this and other threads.)

    Right. But this is all garden-variety human inquiry that is predicated upon human observations and experience.

    >> So even though it is based on empirical evidence, the deduction process itself is NOT direct observation. This is key. There are other methods of arriving at the truth beside direct observation. There is reasonable deduction based on empirically observable facts and established laws of physics, geometry, mechanical engineering, logic, etc.

    Right. I agree with this as long as the deduction remains within the purview of the natural world. That is where these established laws and observations carry weight. Outside of this world, I just do not know if the conclusions derived from empirical laws and observations hold.

    >> Aquinas gave us five proofs of God's existence. These proofs were based on established, observable principles of nature - causation; motion and change; contingency; degrees of perfection; and the repeatable, reliable, tendencies of nature. From these things he made reasonable deductions: A chain of causation must terminate in an uncaused cause; A chain of motion must terminate in an unmoved mover; Intentionality requires a mind; and etc.

    But these deductions are different from the deductions from the crime scene. The conclusions of the logical deductions were empirical phenomena. They stayed within the realm of the natural world, and did not require anything else to understand the accident. I guess what I am trying to say is that not all deductions from empirical observations are created equal, and that those deductions that result in conclusions that are not empirically verifiable should be categorized as speculative at best, despite their deductive nature. I think that is a conservative and cautious approach that will keep us grounded.

    >> These are not "pie in the sky", "brain in a vat", "pretend the real world doesn't exist" philosophical arguments of the kind you often eschew (though those have their place.) No these are real world, empirically based, reasoned deductions that have their roots in fundamental, simple principles that are undeniable.

    You may be correct, but I am skeptical of philosophical deductions that result in metaphysical truths. That just may be my inherent bias, though. There have been too many ideas that were thought to be obviously and inherently true, but when empirically tested were found to be false. The history of science is littered with them, and they all were the result of logical deductions from empirical observations. They also happened to be wrong. So, if the conclusions cannot be empirically tested, then I’m afraid I have to take them with a grain of salt. They may be true, but I prefer to reserve my consent until they can be shown to be operative in the world by more than logical deduction.

    ReplyDelete
  85. >> These, my friend, are akin to the reconstruction of the accident from its remnants - and just as scientific - unless, of course, you exclude such deductions from science. In which case, you'd tie the hands of every forensic team, every archaeologist, every cosmologist - anyone who makes deductions based on existing evidence in order to decipher the truth about what actually happened some time in history (or what's actually happening right now.)

    I can see your point, but I think that metaphysical deductions from empirical observations are different from natural deductions from empirical observations. The latter are based upon the painstaking accumulation of empirical data and the identification of underlying causal patterns that allow a degree of prediction, which allows the deductions that you described above. Take your example of a forensic team. The principles of DNA are well known and replicated and predictable. The principles of bullet analysis are also well known.

    Compare that degree of knowledge to metaphysical entities like an Unmoved Mover. Our reason may be necessarily inclined to draw the conclusion that such an entity must exist, but that may be an inherent defect in our reason when it is pushed to the limits of comprehension. I can trust my reason in the empirical world, because it has an admirable success rate at uncovering empirical truth, but when reason leaves the empirical world, then I become hesitant to trust its conclusions without some additional verification.

    >> So... If you really want to get to the truth, don't exclude reasoned metaphysical deductions like Aquinas'. You may just miss it if you do!

    I’ll try not to, but it is difficult. I just feel like some magic trick has been played on me, and I can’t figure out how it has happened that I have been led, against my judgment, to metaphysical conclusions of a Thomist nature. It is kind of like how most people feel after being exposed to the ontological argument – which I know Aquinas rejected. You just feel like that there is something wrong here, but cannot clearly identify what.

    Now, this may have more to do with my temperament than with anything else, and I may be incorrect to feel this way, but this is where I am right now. Anyway, I am actually looking forward to reading Feser’s books. I have to finish a bit of reading for a presentation, and then will start them. From what I have skimmed, they are intriguing.

    Thanks for the respectful dialogue.

    ReplyDelete
  86. But, actively inquiring is a better way to get knowledge than passively absorbing information by osmosis, because you have no way of evaluating the source of the information when in a passive state.

    "Better" is contextual. I am not "actively inquiring" if I am having the experience of looking at a computer screen right now - I simply am.

    Verification is not an all-or-nothing phenomenon, but is a matter of likelihood, given the evidence. 



    Then that's one interesting definition of "verification". It's not determining that something is, in fact, true (and certainly true at that.) It's arriving at a conclusion based on various evidence, evaluations and assumptions, even if the certain truth is unavailable.

    Your version of verification seems to be something where this is possible: Two people are asked the same question, and give different answers. Both of them can rightly claim that the answer they've given has been verified to be true. Right?

    In fact, let's give an example. Person A tells persons B and C "Write down what time it is right now." B looks at his watch which reads "11:27pm" and writes it down. C looks at his watch which reads "2:17am" and writes it down. Both at the same time. Both believe their answer to be true.

    B verified that the time was 11:27pm. C verified that the time was 2:17am. Correct?

    It is not knowledge until it is verified. The less verification, the more speculation.

    But your "verification" is not "knowing with certainty that something is true". The fact that you're putting "verification" on a slider (something can be 'less verified' or 'more verified') indicates as much.

    So it sounds like it is knowledge after all, eh? It's just knowledge on a slider of some kind - do some verification, get some knowledge.

    ReplyDelete
  87. I was quickly revisiting to copy some links, and I happened to see the following:

    "Just wondering what it is about your argument that makes it rise above faith and dogma, when everyone else's fails to do so. This is an honest question..."

    Is it an honest question, or is it an instance of the logical fallacy of tu quoque? This is an honest question.

    Anyway, as a skeptic and a perspectivist, I think it obvious that my epistemology is more humble and less dogmatic than that of anyone who suggests that there is only one true or correct way to view reality or to determine objective truth, and that somehow little semi-evolved, big-brained primates have succeeded in accomplishing this marvelous feat. Of that, I am skeptical, I freely admit, and so was Nietzsche:

    "Insofar as the word 'knowledge' has any meaning, the world is knowable, but it is interpretable in various ways".

    I've never seen a fact interpret itself. Have you?

    I would add that skepticism does not consist of denying that anything is or can be "real", which seems to be a common mis-definition of the term. A true skeptic simply does not give any particular form of epistemology or inquiry a free pass, or priority of place.

    Given that fact, where, I wonder, do individuals derive the idea that skepticism does not involve belief in anything? Skepticism in whatever flavor involves lack of certainty, and not necessarily lack of belief (or lack of values, for that matter).

    In sum, the issue, to me, is really that of certainty and interpretation. Most of all, the issue is whether any human invention or method, be it religion, philosophy, or science, is equal to the task of interpreting, defining, and delimiting what is "true" and "real". I say that neither is. I would add that dguller, in particular, has much more faith in his perceptual and interpretive mechanisms than I do.

    In any case, if I am wrong. then others are welcome to provide the facts and arguments that refute me. Here is your task, if you want to prove the existence of absolute, objective, a-perspectival truth: To demonstrate truths that are completely invariable, consistently predictable, non-probabilistic, and entirely independent of perspective, consensus, context, or interpretation. And to do this without being tautological or self-referential. Best of luck to you in your attempt, but pardon me if I don't hold my breath while waiting!

    I got what I needed (Edward's links) here, so I am signing off, now. I hope that some find my remarks at least thought-provoking, if not convincing--not that it really matters, of course. What humans do in this universe is of monumental unimportance, on a cosmic scale, and that statement applies a fortiori to what they think, especially when it comes to questions with which they are completely unequipped to deal at this stage of their development. And, of course, that applies to my own views, as well, if that makes anyone feel better. ;-)

    ReplyDelete
  88. Crude:

    >> Your version of verification seems to be something where this is possible: Two people are asked the same question, and give different answers. Both of them can rightly claim that the answer they've given has been verified to be true. Right?

    Yup, especially if their evidence is equally compelling, and there is no way to decide between them. This happens all the time in science. Does increased estrogen cause cancer? The evidence is inconsistent, and so we just do not know at this time what the answer to that question is. Does this bother you?

    >> B verified that the time was 11:27pm. C verified that the time was 2:17am. Correct?

    Not necessarily. What I meant was that verifying that something is true does not result in absolute certainty, because there is always the chance that new data will overturn one’s beliefs. I also meant that there is a hierarchy of evidence where the higher evidence trumps the lower evidence. For example, in medicine, a randomized controlled trial that treatment X works for disease Y trumps a personal observation that treatment X does not work for disease Y. Furthermore, when there is a contradiction between two items of evidence that are equivalent in validity, then looking for additional information would be helpful, if this is possible.

    So, with the example that you gave, there is the same category of evidence (i.e. a watch telling time) that has resulted in a contradiction. How to resolve this contradiction? Well, perhaps one was slow. Checking their watches according to another clock would be a way of reconciling the contradiction. Checking to see if B or C had visual problems would be another. Checking to see if B or C had cognitive difficulties would be another. Checking to see if B or C was deliberately causing trouble to annoy the other would be another. It goes on and on.

    >> But your "verification" is not "knowing with certainty that something is true".

    Right.

    >> So it sounds like it is knowledge after all, eh? It's just knowledge on a slider of some kind - do some verification, get some knowledge.

    Right.

    ReplyDelete
  89. Does this bother you?

    Not really, except for the whole "calling this verification as if it's some special thing seems pointless and misleading." For you it just means having some reason to believe this or that, even if not conclusively. Even making assumptions in the course of the belief isn't strictly forbidden.

    Furthermore, when there is a contradiction between two items of evidence that are equivalent in validity, then looking for additional information would be helpful, if this is possible.

    They don't know there's a contradiction in my example. They just wrote down the time. By your standard, it seems they both have verified knowledge. Even if it's really 2pm.

    ReplyDelete
  90. Crude:

    >> Not really, except for the whole "calling this verification as if it's some special thing seems pointless and misleading." For you it just means having some reason to believe this or that, even if not conclusively. Even making assumptions in the course of the belief isn't strictly forbidden.

    Right, except that it isn't pointless. It is not magical, but it is special, especially when done under the rigorous and methodological constraints of scientific inquiry.

    >> They don't know there's a contradiction in my example. They just wrote down the time. By your standard, it seems they both have verified knowledge. Even if it's really 2pm.

    Sure, given what they individually know, they think they have knowledge. However, there is a problem with your example. You are assuming that their lives have suddenly stopped, but the reality is that after they have checked their watches and recorded their times, they will go on with their day. In that case, it is unlikely that there would be no negative consequences that would provide feedback to them to reassess their beliefs. For example, if any of them had any appointments after checking their watch, then they would be late, and would realize that something was wrong.

    And seriously? You say that it is really 2:00 pm, and they cannot tell that it is daytime outside when their watches say 11:27 pm and 2:17 am? That wouldn’t be a clue to them that something was wrong?

    ReplyDelete
  91. Right, except that it isn't pointless.

    The method itself? Nope, that's useful. It's just average and ordinary. But call it whatever you want. Hell, call it "invoking the eldritch forces of the mighty Cthulhu, long may he feast" if you really wish. I'd think calling it that would be pointless, but hey, the main risk of flair with vocabulary is confusion. And that can be fun, eh?

    Sure, given what they individually know, they think they have knowledge. However, there is a problem with your example. You are assuming that their lives have suddenly stopped

    Yes, I am. All I care about is the fact that right at that moment, by your standards, both of them have "verified" two answers, one of which has to be wrong, both of which can be wrong. I'm interested in here is in fleshing out what you mean by "verification". And hey, we just did that.

    (I do note you're saying 'think they have knowledge' as opposed to really having it, but they do have verification, so now apparently one can verify the truth (except it's not necessarily the truth and verify just means having some reason to believe) but not have knowledge and.. blah blah. Like I said, I wanted to flesh it out, we did that. So, at least there's that.)

    ReplyDelete
  92. Crude:

    >> The method itself? Nope, that's useful. It's just average and ordinary. But call it whatever you want. Hell, call it "invoking the eldritch forces of the mighty Cthulhu, long may he feast" if you really wish. I'd think calling it that would be pointless, but hey, the main risk of flair with vocabulary is confusion. And that can be fun, eh?

    First, you can call it that, if it was true. The reality is, it is not.

    Second, what did I say about scientific inquiry that you feel was an exaggeration?

    >> Yes, I am. All I care about is the fact that right at that moment, by your standards, both of them have "verified" two answers, one of which has to be wrong, both of which can be wrong. I'm interested in here is in fleshing out what you mean by "verification". And hey, we just did that.

    Sure, if you want to do that, then they have verified the time using the observation of checking their watch. And yes, they are wrong. So, we can have good reasons to believe things that are false, which we will only realize when our activity in the world is disrupted by a mistake that is secondary to the false belief. Interestingly enough, by using the information in the false belief to guide our actions, we are given yet another opportunity to verify or falsify the belief. So, it is an ongoing process that does not just stop, and it falls short of full certainty.

    ReplyDelete
  93. Anonymous:

    >> The stick in the water does not exist as bent only in relation to how you perceive it when you take it out. But how do you know that the illusion isn't the way you experience the stick when it's out of the water and not in it? In fact, how have you come to think that what is contrary to your (or every day) 'regular expirience' is what is illusory? It could be the case that the stick is really bent, but when it's out of the water, something causes (and distorts) it to appear as straight.

    Sorry, I missed this remark earlier, and wanted to comment on it.

    Your reasoning would work, except for a few things, all of which are empirical.

    One, when I enter the water, I DO NOT BEND.

    Two, when I hold the stick in my hand under water, it is NOT BENT.

    Three, other people, when they enter the water, do not report suddenly feeling their bodies bend.

    The best inference is that there is something about the water that distorts our perception, and when we study the properties of light as it passes through water, then we understand how this illusion is possible. There is nothing earth shattering happening here that requires us to radically revise all our previous beliefs about the world. There is only an odd empirical phenomenon that can be understood by empirical methods of inquiry. There is no need to bring in philosophical speculation at all.

    ReplyDelete
  94. Kevin:

    >> Here is your task, if you want to prove the existence of absolute, objective, a-perspectival truth: To demonstrate truths that are completely invariable, consistently predictable, non-probabilistic, and entirely independent of perspective, consensus, context, or interpretation. And to do this without being tautological or self-referential. Best of luck to you in your attempt, but pardon me if I don't hold my breath while waiting!

    First, I doubt that anyone disbelieves that the planet earth only began to exist once human beings attained conscious awareness of it. It exists independently of our perceptual apparatus and cognitive capacities, and will continue to do so long after we have gone extinct.

    Second, I think that you set the standard impossibly high. You might as well ask me to describe something without using language or words, and then when I cannot do so, you win … what? I mean, does that somehow refute that something exists independently of us? Does the fact that we have to use the word “dog” and the concept “dog” to describes dogs imply that dogs themselves are artifacts and unreal? Good luck coming up with an argument to show that.

    Third, if perspectivism is true, then it must be true independent of your perspective, which would be self-refuting. Fortunately, we can believe in the independent existence of many entities, which we happen to experience from our particular perspectives. There is no contradiction in this, unless you assume from the outset that we are trapped within our minds and subjectivity, which is fundamentally divorced from an outer and external world. Luckily there is no need to believe this at all.

    ReplyDelete
  95. Kevin,

    My question was honest insofar as, if I am sunk in faith and dogma, how can I tell? I didn't think I was sunk in faith and dogma; you come along and announce that I am. How do I know that by believing you, I am not merely swapping one dogma for another? Especially given that the premise of your announcement is that I can't tell the difference between faith and dogma and true rationality; for otherwise, I would have figured out my true situation on my own.

    I'm not sure how to understand you. When you write that your view "is more humble and less dogmatic than that of anyone who suggests that there is only one true or correct way to view reality or to determine objective truth", are you saying that thinking of your view as humbler and less dogmatic than others is the one true and correct way to consider your view? Or is there no objective truth about your view, in which case I may just as well go on thinking of it as actually more arrogant and false than others?

    ReplyDelete
  96. dguller: "But these deductions are different from the deductions from the crime scene. The conclusions of the logical deductions were empirical phenomena."

    No the conclusions are not "empirical". The conclusions are extrapolated ideas about what happened. The actual event has passed, the truth is what we're after and the only way to find it is to examine evidence and extrapolate from that. The evidence is empirical, the extrapolation and conclusions are not.

    "Outside of this world, I just do not know if the conclusions derived from empirical laws and observations hold."

    No one knows this for sure. But reason is reason. It's a darn sight better than irrationality!

    "The principles of DNA are well known and replicated and predictable. The principles of bullet analysis are also well known. Compare that degree of knowledge to metaphysical entities like an Unmoved Mover."

    Apples and oranges. You are comparing, on the one hand, what's known about DNA and bullets, to a conclusion: an unmoved mover, on the other.

    'Apples to apples' would be to compare what's known about DNA and bullets to what's known about the principles of motion or causation. And 'oranges to oranges' would be to compare the conclusion that the bullet was fired from gun X to the conclusion that a chain of motion must terminate in an unmoved mover.

    "I can trust my reason in the empirical world, because it has an admirable success rate at uncovering empirical truth, but when reason leaves the empirical world, then I become hesitant to trust its conclusions without some additional verification."

    What if the empirical world IS the verification you seek for the existence of the supernatural? That's pretty much what Aquinas was saying - if you take a close, reasoned look at what goes on in this world, you'll soon discover that there needs to be something NOT OF THIS WORLD in order for everything we see to exist and function.

    "Thanks for the respectful dialogue."

    Same to you. I wish you well.

    ReplyDelete
  97. Daniel:

    >> No the conclusions are not "empirical". The conclusions are extrapolated ideas about what happened. The actual event has passed, the truth is what we're after and the only way to find it is to examine evidence and extrapolate from that. The evidence is empirical, the extrapolation and conclusions are not.

    Yes, but the idea that was concluded is about something empirical. It is not about something metaphysical or supernatural. That is what I meant. In other words, we are still operating within the empirical world, according to well-established natural laws and regularities.

    >> No one knows this for sure. But reason is reason. It's a darn sight better than irrationality!

    True, but I think it would more humble and prudent to admit that beyond the empirical world, even if our deductions are logical, we just do not know for sure if they are true. There is an underlying assumption to metaphysics that the rules that apply within the empirical universe also apply outside of it, and this is far from clear, at least to me.

    >> Apples and oranges. You are comparing, on the one hand, what's known about DNA and bullets, to a conclusion: an unmoved mover, on the other.

    But that is the point. You are inferring from the fact that we can make legitimate inferences from the evidence of a crime scene (which is all within the natural world) to the fact that we can make legitimate inferences from the empirical world to a metaphysical one (which is not within the natural world). As you said, apples and oranges. So, why did you make the original comparison?

    ReplyDelete
  98. Daniel:


    >> What if the empirical world IS the verification you seek for the existence of the supernatural? That's pretty much what Aquinas was saying - if you take a close, reasoned look at what goes on in this world, you'll soon discover that there needs to be something NOT OF THIS WORLD in order for everything we see to exist and function.

    I would say that there is a difference between being compelled to believe something is true and something being true. I agree that metaphysical arguments of the Thomistic variety can be quite persuasive, and seem inevitable and incorrigible, but that may say more about our psychology than about reality itself.

    I have always found the Enlightenment vision of keeping reason within proper limits as compelling. We know that reason works within the empirical world. We have an abundance of evidence that this is the case. However, we have no evidence that reason also works with equal success outside the empirical world. It might, but we have no way of knowing. So, rather than engage in endless speculation about something that we lack sufficient knowledge of, we should declare that we just don’t know, and focus our resources on activities that have a better chance of reaching the truth.

    Any thoughts?

    ReplyDelete
  99. dguller:

    "Outside of this world, I just do not know if the conclusions derived from empirical laws and observations hold."


    Daniel Smith:
    "No one knows this for sure. But reason is reason. It's a darn sight better than irrationality!"

    dguller:
    "True, but I think it would more humble and prudent to admit that beyond the empirical world, even if our deductions are logical, we just do not know for sure if they are true. There is an underlying assumption to metaphysics that the rules that apply within the empirical universe also apply outside of it, and this is far from clear, at least to me."

    ***

    A naturalist (and most atheists are naturalists) makes assumptions — has to make them — that go beyond observable space-time just like the theist. I have written about this in chapter 2 “The origin of the universe: eternal God, eternal matter or eternal fields” of my article:

    http://home.earthlink.net/~almoritz/cosmological-arguments-god.htm

    My conclusion there is that:
    “In fact, in order to be able to believe in a naturalistic origin of the universe, the atheist must *negate data* on what observational science tells us about actual matter, energy and fields, and instead believe in miraculous properties of such entities that science has not shown to exist.”
    (For details on why, see the article.)

    Ironically, the problem with eternal matter that I describe there throws us right back to the argument from the Unmoved Mover.

    ***

    So if dguller says "Outside of this world, I just do not know if the conclusions derived from empirical laws and observations hold" then this is precisely a crucial problem for the atheist. He must negate what holds inside our universe when it comes to a wider universe that supposedly can generate ours.

    And when someone like dguller claims you should only accept things from empirical evidence, then atheism is out the window too, since the origin of the universe is in principle outside of the scope of observational science, due to the cosmological particle horizon. The default position in that case is agnosticism (literally "I don't know"), not atheism, and also not "agnostic atheism", since that position already attributes probabilities where a simple "I don't know" should stand in its place.

    ReplyDelete
  100. Al:

    >> So if dguller says "Outside of this world, I just do not know if the conclusions derived from empirical laws and observations hold" then this is precisely a crucial problem for the atheist. He must negate what holds inside our universe when it comes to a wider universe that supposedly can generate ours.

    No, I do not have to make a positive claim at all about what happens outside the universe, if this even makes sense. All I have to say is that I do not know what happens there, and more importantly, neither do you.

    Now, I can certainly speculate about what happens, staying within various assumptions, but whatever the conclusions are must be held to be tentative, because without confirmation that there is a symmetry between what happens within the universe to what happens outside the universe, it is all speculation.

    >> And when someone like dguller claims you should only accept things from empirical evidence, then atheism is out the window too, since the origin of the universe is in principle outside of the scope of observational science, due to the cosmological particle horizon. The default position in that case is agnosticism (literally "I don't know"), not atheism, and also not "agnostic atheism", since that position already attributes probabilities where a simple "I don't know" should stand in its place.

    Right. But you forget one important detail. The atheist does not stake his life upon what occurs outside of space-time. The theist does. The theist makes a variety of positive claims about how what happens outside of space-time is fundamentally important to how we live our lives within space-time, and claims to know how we should think, feel and behave on the basis of these speculations. It is the theist that is making the positive claim here, and all an atheist has to say is that the theist has no way of knowing whether what he or she believes is actually true.

    It is insufficient for the theist to then reply that the atheist does not know either. That does not automatically add warrant to the theist’s position at all. It just means that we are all in the same fog together and lack any clear compass to find our way.

    A more important question is whether one should base the foundations of one’s life upon beliefs that are utterly without foundation.

    ReplyDelete
  101. dguller,

    "I have always found the Enlightenment vision of keeping reason within proper limits as compelling. We know that reason works within the empirical world."

    Are you familiar with the works of Immanuel Kant? Keeping reason within proper limits was his lifetime project, and is no simple thing, as the earlier Enlightenment philosophers (e.g. Hume) naively thought.

    ReplyDelete
  102. Dmt117:

    >> Are you familiar with the works of Immanuel Kant? Keeping reason within proper limits was his lifetime project, and is no simple thing, as the earlier Enlightenment philosophers (e.g. Hume) naively thought.

    Yup, but you have to remember that he began his project from a Humean framework, which presupposed the truth that all we experience is subjective mental states that are fundamentally divorced from the world. Remeber that Hume woke him from his "dogmatic slumber". He made a Herculean effort to reconnect our internal representations to an external reality, but fundamentally generated more paradoxes and complications. Without that initial assumption, there is no need for his many contortions. Furthermore, it becomes more straightforward to keep our reason within limits. I mean, we typically do this on a regular basis in which we defer commenting on a matter when we lack sufficient expertise in that area. I say that we should do the same thing about metaphysical matters.

    ReplyDelete
  103. Al:

    And one more thing. Remember, my contention is that none of us really knows with any degree of certainty what laws and regularities are operative outside of space-time. Now, there are a number of individuals who make positive claims about what is going on outside of space-time and how these entities and events impact our daily lives. If all of us are ignorant, then how can one make positive claims at all, and how can one decide between them?

    Say that there is an uncharted island in the ocean that no-one has visited. A claims that the people there are compassionate to their children, and B claims that they are cruel to their children, and C claims that they use both styles. What can an independent observer of this discussion say, except that without actually having any knowledge of the people on the island, neither A, B nor C can claim what they do with any degree of certainty. Notice that this observer did not have to visit the island themselves to make this claim, but only to point out that those with positive claims lack the warrant to make them.

    Similarly, say that A says that X is outside space-time, and B says that Y is outside space-time, and C says that Z is outside space-time. Neither A, B, nor C have the requisite knowledge to claim that X, Y or Z occurs outside space-time. All an a-Xist, a-Yist, and a-Zist needs to say is that no one knows what they are talking about and can reject X, Y and Z as viable options. Notice that this does not require the a-Xist, for example, to have positive knowledge about what occurs outside of space-time.

    ReplyDelete
  104. dguller,

    I think Kant has a little more significance than you give him credit for. It is true that he was inspired by Hume, but his project of delimiting the "proper limits" of reason transcends Hume.

    Like you, Kant wished to confine reason to "empirical" limits. But he saw that defining and justifying exactly what those limits are is a problematic project. For the process of justifying those limits is not itself an act of empirical reason, otherwise the whole project is circular. Thus his appeal to "pure reason."

    We can't dodge Kant by appealing to "direct experience", as in "we directly experience causal regularities", because just what constitutes direct experience is exactly at issue. Hume denied that we directly experience causal regularities. On the other hand, Aristotelians claim that we directly experience the foundational elements of classical philosophy. We may or may not disagree with one or the other of these people, but the point is what constitutes "direct experience" is the decisive question.

    ReplyDelete
  105. Dmt117:

    >> Like you, Kant wished to confine reason to "empirical" limits. But he saw that defining and justifying exactly what those limits are is a problematic project. For the process of justifying those limits is not itself an act of empirical reason, otherwise the whole project is circular. Thus his appeal to "pure reason."

    I disagree. I think that it is empirical. You observe that by focusing your attention to understanding the world using scientific methodology, for example, your knowledge of the world grows by leaps and bounds, and results in technological innovation that allows you to do more than you ever thought possible and learn more and more. You also observe that by focusing your attention to understanding a metaphysical reality beyond space-time results in confusion, lack of consensus, no real acquisition of new knowledge beyond the original assumptions made, and no real way to confirm whether any conclusions made are really true. You also observe that the latter assumes from the beginning that the principles and regularities that are operative within space-time are also operative outside of space-time, which we do not observe at all. I think that given those observations it is fair to conclude that we should focus our energy towards the former and not the latter, if we are serious about understanding ourselves and our world.

    >> We can't dodge Kant by appealing to "direct experience", as in "we directly experience causal regularities", because just what constitutes direct experience is exactly at issue. Hume denied that we directly experience causal regularities. On the other hand, Aristotelians claim that we directly experience the foundational elements of classical philosophy. We may or may not disagree with one or the other of these people, but the point is what constitutes "direct experience" is the decisive question.

    I think that we experience regularity in a direct way. I see that the sun rises every day in a predictable fashion, for example. It is in this regular meeting of my expectations of what will happen that law-like phenomena are experienced. And actually what Hume denied was that we directly experience NECESSARY causal regularities in nature. Contingent causal regularities were never a problem, as far as I recall.

    And as for what counts as “direct experience”, it is what we experience with our senses about the world. I don’t think that this is particularly controversial, except when one approaches the issue with philosophical baggage that ultimately distorts our experience to be more suited to the system that one adheres to. Naturally, paradox and confusion follow.

    ReplyDelete
  106. dguller,

    Yes, I know you disagree. That's the point. We have a disagreement about what constitutes valid empirical reason. Simply asserting what you believe to be valid empirical reason doesn't resolve the dispute; it just defines our differences.

    "And as for what counts as “direct experience”, it is what we experience with our senses about the world."

    I agree, as would St. Thomas Aquinas. And as a Thomist, I say that what I experience with my senses are beings undergoing various modes of substantial and accidental change. If you don't agree with me - and I think you don't - then this disagreement proves that the issue is controversial.

    Modern philosophers all do the same thing: They claim to found and restrict their thought to "direct experience", and thereby show that everyone before themselves was lost in metaphysical illusions. Descartes did it, Hume did it, Locke did it, Kant did it, etc., etc. Of course, they all have different conceptions of what constitutes "direct experience", so they disagree with each other. What is so frustrating about modern philosophers is that, when the next guy comes along, he simply makes a new assertion concerning the content of "direct experience" and goes from there, rather than addressing the root of the controversy, which is the meaning and content of direct experience itself.

    ReplyDelete
  107. dmt:

    >> Yes, I know you disagree. That's the point. We have a disagreement about what constitutes valid empirical reason. Simply asserting what you believe to be valid empirical reason doesn't resolve the dispute; it just defines our differences.

    Okay, fair enough.

    >> I agree, as would St. Thomas Aquinas. And as a Thomist, I say that what I experience with my senses are beings undergoing various modes of substantial and accidental change. If you don't agree with me - and I think you don't - then this disagreement proves that the issue is controversial.

    If you mean what I think you mean, then I don’t disagree.

    >> Modern philosophers all do the same thing: They claim to found and restrict their thought to "direct experience", and thereby show that everyone before themselves was lost in metaphysical illusions. Descartes did it, Hume did it, Locke did it, Kant did it, etc., etc. Of course, they all have different conceptions of what constitutes "direct experience", so they disagree with each other. What is so frustrating about modern philosophers is that, when the next guy comes along, he simply makes a new assertion concerning the content of "direct experience" and goes from there, rather than addressing the root of the controversy, which is the meaning and content of direct experience itself.

    Sure, and that is another fair point. I would add that what the modern philosophers did was to utilize fallacious arguments to justify their picture of the human mind in which it only directly experienced subjective mental states. Remember, this was not a claim that was just asserted out of the blue, but based upon arguments from illusion, dreaming, an evil genius, and other things, that they thought justified their model of the mind. Again, these arguments are either fallacious or are rooted in the sheer possibility of philosophical fairy tales. So, there is no reason to buy into their model of the mind at all, and since that model is what led to centuries of philosophical confusion, then we are better off without it.

    Having said that, let us focus upon my particular claims, and not what previous philosophers have believed. If we can agree upon what counts as direct experience, then how do you justify going beyond direct experience into a realm beyond space-time and claim to have knowledge about it? Does that not assume that the regularities and principles that operate within your experience also are operative beyond the empirical universe?

    ReplyDelete
  108. dguller,

    OK, those are fair points and I will try to answer your question. I will start by pointing out that I don't agree with the implications of this sentence:

    "If we can agree upon what counts as direct experience, then how do you justify going beyond direct experience into a realm beyond space-time and claim to have knowledge about it?"

    The implication is that direct experience is thoroughly conditioned by space-time, and that it is reasonable to contrast this direct experience with a "realm beyond" it about which we allegedly can't have any knowledge. This is not a distinction that was made by the classical philosophers and was, in fact, introduced by modern philosophers. The principle that experience is thoroughly conditioned by space-time is the foundation stone of Kant's attack on traditional metaphysics (his "transcendental aesthetic"). Now I agree with you that if we accept Kant's transcendental aesthetic, then metaphysics as traditionally practiced is impossible.

    But I deny that our experience is thoroughly conditioned by space-time. I think we all have direct experience that transcends space and time. For instance, I experience football fields, notebook paper and envelopes all as "rectangular." This "rectangular" that I experience is, I claim, not conditioned by time and space ("rectangular" isn't one thing over here and another over there, and doesn't change day to day). Kant would deny this, of course, and say that "rectangular" is merely one of the categories my mind places on experience. But, if I understand you at this point, you don't agree with Kant on this. The "rectangular" we experience is not something in our heads but a true statement about certain things as they really are.

    So the first step is to close the false distinction made in experience between so-called real "space-time" experience and a mystical experience beyond space-time. I'll stop here, because if you don't agree that we actually have experience not conditioned by space-time, there is no point going on.

    ReplyDelete
  109. dguller: "There is an underlying assumption to metaphysics that the rules that apply within the empirical universe also apply outside of it."

    I would say that this is the crux of your objection to metaphysical thought - correct?

    Let me just say that I don't think most metaphysical arguments go as far as what you are saying. I don't think any claims are made that the properties of this world can be expected to work outside space/time. What they are saying is that the properties we observe within this world do not make sense unless there IS something working from outside nature.

    So the arguments point to an "external" being and an "external" realm but make no claims as to how that being or realm operate.

    Theologians will tackle those questions but philosophers (from my limited knowledge anyway - I could be wrong) don't usually cross that threshold.

    Aquinas said that (paraphrasing) reason tells us there is a God, revelation tells us how he works.

    Of course you can always use your "empirical verification = experience" method of investigation and ask God to reveal himself to you. If you do that diligently - with an honest and open heart - I'm confident that you too will join the billions of us who have experienced for ourselves the reality of God.

    Those are my thoughts. Thanks for listening.

    ReplyDelete
  110. Dmt:

    >> But I deny that our experience is thoroughly conditioned by space-time. I think we all have direct experience that transcends space and time. For instance, I experience football fields, notebook paper and envelopes all as "rectangular." This "rectangular" that I experience is, I claim, not conditioned by time and space ("rectangular" isn't one thing over here and another over there, and doesn't change day to day).

    Wouldn’t it be easier to say that you experience individual rectangles in space-time? Personally, I doubt that you experience the abstract mathematical concept of rectangle in your experience. After all, the ideal rectangle that is abstracted from experience consists of perfectly straight lines at perfect right angles. Such perfection just does not exist in the empirical world. Or are you saying that you actually experience perfect rectangles when you observe rectangular objects in the world?

    Another way out of this would be to say that we do experience rectangles, but that the concept of rectangle that we are using to identify them as rectangles is imperfect and is basically “two roughly straight lines at roughly right angles, which are of unequal length”. I will grant that you experience these types of rectangles, but so what? Of course these rectangles exist in the world. To establish your claim you would have to show that you experience perfect rectangles in your experience, because only perfect rectangles are outside space-time, usually in some Platonic mathematical fantasy land.

    ReplyDelete
  111. Daniel:

    >> What they are saying is that the properties we observe within this world do not make sense unless there IS something working from outside nature.

    But remember that this entire idea is modeled on what we experience inside nature. It is inside nature that we experience one thing causing another. We then infer that there must be something outside nature that is causing nature. I think that we are applying a concept that derives its sense within nature to something outside nature.

    Believe me, I can see just how intuitively obvious this argument is. I mean, everywhere we look we see potential becoming actual when a pre-existing actual entity acts in such a way to actualize the potential. It certainly seems reasonable that something purely actual must underlie the entire process, and I can feel the pull and force of the argument. But then I am reminded of the fact that the conversion from potential to actual occurs “everywhere we look”, which is necessarily within the universe. To say that this principle must equally be applicable outside the universe just makes me suspicious. I begin to wonder how someone could know this, and I just do not know how.

    >> So the arguments point to an "external" being and an "external" realm but make no claims as to how that being or realm operate.

    Fair enough.

    >> Of course you can always use your "empirical verification = experience" method of investigation and ask God to reveal himself to you. If you do that diligently - with an honest and open heart - I'm confident that you too will join the billions of us who have experienced for ourselves the reality of God.

    Believe me, I have tried. There was a time when I was devout and religious. It just never worked for me, and I simply lost my faith in a particular God, but still adhered to something higher and beyond, along the lines of deism, but even that was lost over time. It just became too hard to believe.

    But thank you for your recommendation. It is appreciated.

    ReplyDelete
  112. dguller,

    It would be easier to say that I experience individual rectangles in spacetime, but it doesn't seem helpful to do so. What is it about all these individual rectangles that justifies calling them "rectangles?"

    That is why I wrote that I experienced "rectangular" things rather than "rectangles." I was emphasizing that all these objects share something in common - rectangularness. And it is a matter of direct experience that they do. Where is this common feature located? It's got to be either in our heads (Kant) or in the things themselves (Aristotle), from which we abstract the "perfect" rectangles of which you speak. In other words, objects may participate more or less perfectly in the principle of "rectangular", but whether they do or not, the very fact that several objects share the same principle to any degree at all shows that the principle itself transcends time and space. The only way to avoid this is to deny that the principles exist commonly in things themselves, and we are on our way to Kant.

    I don't propose a Platonic fantasy land (actually, I don't think Plato did either). I propose the Aristotelian hylemorphic understanding of nature, under which our experience is not artificially forced into the categories of space and time, which can't do justice to it.

    ReplyDelete
  113. Dmt:

    >> In other words, objects may participate more or less perfectly in the principle of "rectangular", but whether they do or not, the very fact that several objects share the same principle to any degree at all shows that the principle itself transcends time and space.

    How does this follow? Whether “rectangular” exists in the mind or in the objects, both exist within space-time.

    And why cannot “rectangular” exist in BOTH the mind and the object, but in different senses. In the mind, it exists as a concept and idea, and in the object as a quality. And again, our concepts and ideas occur within space-time, as do the objects that they refer to.

    ReplyDelete
  114. dguller,

    Yes, rectangular exists both in the mind and in things, in different modes. This is Thomism. The key point is that it is the same thing that exists both in the mind and in things.

    An important thing to remember is that I'm not talking about something that exists beyond spacetime, as though it has no reality in spacetime. I'm talking about things that transcend spacetime. In a physical analogy, the Mediterranean transcends the Adriatic, not because it exists in a different place altogether, but because it encompasses it but is more than it. Similarly, I'm saying that "rectangular" transcends spacetime not because it has nothing to do with spacetime, but because our experience of it cannot adequately be captured in the categories of space and time.

    For instance, "rectangular" exists both in things and in the mind. And it exists completely in the mind and completely in things. The principle of "rectangular" is not divided into bits and spread out amongst all rectangular things, and all minds. You don't have half the principle and I have the other half. We both have the entire principle completely. There is no way such a state of affairs can be understood in the categories of spacetime. Something can't both completely be here and completely be there in spacetime; it can only partly be here and partly be there. So our experience - our everyday experience - transcends the categories of spacetime.

    ReplyDelete
  115. Dmt:

    >> Yes, rectangular exists both in the mind and in things, in different modes. This is Thomism. The key point is that it is the same thing that exists both in the mind and in things.

    Why do you think that the same thing exists in both the mind and in things? Why does there have to be a third entity that mediates between the concept in the mind and the quality of the object? Why can’t it just be true that the concept “rectangular” refers to rectangular objects?

    Here’s another way to look at it. Take the typical Tarski formulation of a truth claim: a proposition “p” is true iff p. In other words “It is raining” is true if and only if it is raining. That’s all there is to it. However, it is tempting to look for some third thing between “p is true” and “p” – such as Truth – that mediates the relationship, but the reality is that there is nothing like that going on, and trying to find something will just muddle things.

    >> I'm talking about things that transcend spacetime. In a physical analogy, the Mediterranean transcends the Adriatic, not because it exists in a different place altogether, but because it encompasses it but is more than it. Similarly, I'm saying that "rectangular" transcends spacetime not because it has nothing to do with spacetime, but because our experience of it cannot adequately be captured in the categories of space and time.

    First, if “rectangular” includes perfect lines, perfect angles, and other abstract qualities, then we do not experience it at all, unless your definition of “experience” includes “intellectual apprehension”. And if that is what you are saying, then do we also experience infinity, because we can intellectual apprehend, and therefore experience, such a concept?

    Second, I can totally understand your physical analogy that explains transcendence. But remember, that is only because it is PHYSICAL, and therefore, empirical and experiential. It occurs within space-time. When you talk about X transcending space-time, the implication is that X “encompasses” space-time, but “encompasses” is a word that is also operative in space-time. So, you are using a term that gets its very meaning in the context of the relationship of entities within space-time, and now are trying to use it outside that context to refer to something that is “more than” space-time.

    Third, there is an ambiguity in what you say about “rectangular”. There is the ideal mathematical meaning that involves perfect lines, perfect right angles, and so on, and then there is the mundane everyday meaning that involves roughly straight lines and roughly right angles, and so on. I agree that we experience the latter, but I disagree that we experience the former. And why do we have to experience the ideal in order to experience the mundane? Why can’t it go the other way? In that way, we could experience rectangular objects by using the everyday concept of rectangular, and then the mathematical concept of rectangular is just a further abstraction from that. However, that further abstraction is removed from our experience, because it is a perfect abstraction that does not exist in the world.

    ReplyDelete
  116. Dmt:

    >> For instance, "rectangular" exists both in things and in the mind. And it exists completely in the mind and completely in things. The principle of "rectangular" is not divided into bits and spread out amongst all rectangular things, and all minds. You don't have half the principle and I have the other half. We both have the entire principle completely. There is no way such a state of affairs can be understood in the categories of spacetime. Something can't both completely be here and completely be there in spacetime; it can only partly be here and partly be there. So our experience - our everyday experience - transcends the categories of spacetime.

    Again, I do not think that we need to postulate a third entity that mediates between rectangles in the world and our concept of rectangular. A minimalist account should be just fine, I think. We have concepts that accurately represent objects in the world, if objects in the world have the characteristics of our concept. There is no third metaphysical and mathematical template that we are accessing in order to make sense of our use of concepts in the world.

    Remember, we do not have to reify everything into ontological entities. When I use the sentence, “No-one is coming”, it does not imply that there is something that I call “No-one” that is coming. It just means that there is actually no person at all that is coming. There is no need to postulate a metaphysical No-one to explain the sense in that sentence.

    ReplyDelete
  117. Dmt:

    And one more thing about your use of “transcendent”.

    There are two senses of this word. One is the one that you use in which if X transcends Y, then X and Y are still related by virtue of having a common underlying substance, but X is just greater than Y in some way. So, your example of the Mediterranean transcending the Adriatic would work, because both the Mediterranean and the Adriatic are oceans. The other is the one that is typically used in which if X transcends Y, then X is actually ontologically separate and beyond Y. So, when someone says that God transcends the universe, then it is understood that he and the universe do not share a common underlying substance between them.

    If you are claiming that mathematical concepts transcend space-time in the first sense, then what is the common substance that mathematical concepts and space-time share? And what is “greater” about mathematical concepts in this context?

    And if you are claiming that mathematical concepts transcend space-time in the second sense, then we do not experience them at all, because their perfect qualities are never experienced in the world, because the world is imperfect and imprecise, and is a rough approximation that does not meet the rigorous standards of mathematics. Remember that that was the basis of the ancient preoccupation with mathematics, that they were a doorway through which one could enter an immutable and perfect world away from the mutable and imperfect empirical world. Plato’s cave captures this desire well.

    ReplyDelete
  118. dguller,

    "A minimalist account should be just fine, I think. We have concepts that accurately represent objects in the world, if objects in the world have the characteristics of our concept."

    I will answer your question about the meaning of transcendent in a my next post. I don't find your account minimalist, because you've added a third entity, to wit, "characteristics" of a concept. Are characteristics of concepts themselves concepts? If so, then I don't see how your account is different than mine, because you say that objects in the world "have" the characteristics of the concept in our mind. So rather than sharing concepts, objects and our mind share characteristics which, I suspect, are really just other concepts. Why not eliminate these extra entities and just say at the start that my mind and objects can share the concept rectangular?

    ReplyDelete
  119. As far as your options with respect to the meaning of transcendent, I think the first option is closer to the truth. I subscribe to the Aristotelian hylemorphic understanding of nature, wherein commonly experienced beings are composites of form/matter, e.g., I am a composite of matter and the form man. This form exists across men and also in our minds, but it is the same form in all cases (although, as you point out sometimes in different modes), and does not exist as a "third entity" above men and minds.

    So there is not some third substance that unites the concept of "rectangular" in my mind and the "rectangular" that exists in objects. "Rectangular" just is that common substance between my mind and things. (Actually, your "characteristics" sound more like an attempt at a third substance mediating between things and the mind.)

    ReplyDelete
  120. I notice I didn't answer your question about what is greater in the context of the first option with respect to the meaning of transcendent.

    What is "greater" about "rectangular" under the hylemorphic understanding is that the form "rectangular" - the one shared across minds and objects - cannot itself be understood in the categories of spacetime. It can be used to understand things in spacetime, but is not itself reducible to the categories of spacetime. The form "rectangular" is both complete and whole in my mind and complete and whole in your mind, and it is identically the same form: This is a reality that makes no sense under the categories of spacetime. Thus, insofar as forms are shared as universals, they transcend spacetime.

    ReplyDelete
  121. Dmt:

    >> I don't find your account minimalist, because you've added a third entity, to wit, "characteristics" of a concept. Are characteristics of concepts themselves concepts?

    First, I do not think that “characteristics” is an entity at all. Is the characteristic or property “red” an independent entity? It is very appealing to reify “red” into some thing, but there is no need to do so. The bottom line is that this concept gets its meaning from our practices and rule-governed behavior, which we learn from childhood in a society of human beings that share a common set of cognitive and behavioral capacities. “Red” is just the word that we use when identifying red objects in the world, which is something we learn how to do without any paradox or problem.

    Second, I haven’t added anything new at all. Yes, I used a new word, but remember that we are dealing with a circular set of concepts that all relate to one another. There is no foundational concept from which all the others derive their meaning. So, when I use a new word within the context of this circular set of concepts, I haven’t actually added anything new, which maintains the minimalist nature of what I am proposing.

    Third, what do you mean by “characteristics of concepts”? One characteristic of a concept is that it is perceivable by a mind, which in one sense is a concept, but in another sense is a real event in the world.

    >> If so, then I don't see how your account is different than mine, because you say that objects in the world "have" the characteristics of the concept in our mind. So rather than sharing concepts, objects and our mind share characteristics which, I suspect, are really just other concepts. Why not eliminate these extra entities and just say at the start that my mind and objects can share the concept rectangular?

    Again, you are assuming that there is a thirty entity, “characteristics” that exist within concepts and objects, and that somehow tie the two together. I get the impression that your model of how the mind represents the world is that analogous to a cookie-cutter, which has assumed a certain shape. This shape is present in a thought and is also present in an object in the world, and what they have in common is that they have been imprinted by this cookie-cutter, which is an independently existing entity. You seem to think that without this external cookie-cutter, there would be no common shape.

    But why think that this is how it works?

    ReplyDelete
  122. dmt:

    >> As far as your options with respect to the meaning of transcendent, I think the first option is closer to the truth. I subscribe to the Aristotelian hylemorphic understanding of nature, wherein commonly experienced beings are composites of form/matter, e.g., I am a composite of matter and the form man. This form exists across men and also in our minds, but it is the same form in all cases (although, as you point out sometimes in different modes), and does not exist as a "third entity" above men and minds.

    Okay. I’m actually okay with that.

    >> So there is not some third substance that unites the concept of "rectangular" in my mind and the "rectangular" that exists in objects. "Rectangular" just is that common substance between my mind and things. (Actually, your "characteristics" sound more like an attempt at a third substance mediating between things and the mind.)

    Yes, but “rectangular” is not a substance at all, and does not exist “between my mind and things”. Again, your way of putting things adds a third substance, “rectangular”, between “my mind” and “things”, that they both participate in somehow. And this only becomes a problem if one tries to treat Aristotle’s forms as entities themselves, which is based upon the assumption that all conceptual content must refer to independently existing substances, which is unnecessary and confusing. And it leaves one struggling to explain how the mind and objects in the world both share this third entity, where it happens to exist, and leaves us with mathematical entities that independently exist, which results in its own set of confusions.

    I think that this is a classic example of forgetting how we actually act in the world, and try to impose a particular philosophical theory upon our experience. Once you start with the assumption that anything that exists is some kind of substance, then you are stuck trying to identify bizarre entities and how they relate to us and the world, such as “nothing” and “2 + 2 = 4” and “rectangular”.

    However, when you just look at what we do in the world, you find a much simpler picture. We see rectangular objects, and learn to call them “rectangular”, and learn the properties of “rectangular”, which allows us to use that term to refer to other rectangles that we come into contact with. How do we know if we are using the word “rectangular” correctly? Well, we use it, and see the consequences in terms of the behavior of other language users. If I call a triangle a rectangle, then someone will point out that I am misusing the term. And that will be that.

    Again, I can certainly feel the pull to reify our concepts into existing entities. I do struggle against it sometimes, because it is so obvious. But this may be more due to the fact that our language is insufficiently fine-grained to capture all the features of our world, and so we use the same word to cover multiple things that differ in important ways. So, maybe “exist” is too general a term, especially when it applies to objects, concepts, properties, numbers, possibility, actuality, and so on. Sure, all those things – see I can’t help but use the word! – are real in some sense, but in different ways. The possibility is real, but not in the same way that actuality is real, and it would be a category mistake to treat them in the same way.

    I think that what your type of metaphysics does is to keep looking for substances that are affecting one another, and that “affecting one another” becomes another substance, and it all just gets out of control. We need to pay attention to the differences and exceptions to this drive in order to keep it in check. That would save us a lot of headaches.

    ReplyDelete
  123. dguller,

    "The bottom line is that this concept gets its meaning from our practices and rule-governed behavior, which we learn from childhood in a society of human beings that share a common set of cognitive and behavioral capacities. “Red” is just the word that we use when identifying red objects in the world, which is something we learn how to do without any paradox or problem."

    Well, the question at issue isn't how we become educated into the use of the world "red". It's the philosophical question of the relationship between our concepts and reality. Of course, we can get along through life without ever asking this question, which is why most people aren't philosophers. So we can, if we choose, just stop at the fact that we use the word "red" to identify red things, and avoid all the problems and paradoxes that arise when we wonder how it is we are justified in attributing the single concept red to a variety of different things. Fair enough. But affirming that we can get along without addressing philosophical questions is not the same as answering them.

    ReplyDelete
  124. dmt:

    >> What is "greater" about "rectangular" under the hylemorphic understanding is that the form "rectangular" - the one shared across minds and objects - cannot itself be understood in the categories of spacetime. It can be used to understand things in spacetime, but is not itself reducible to the categories of spacetime. The form "rectangular" is both complete and whole in my mind and complete and whole in your mind, and it is identically the same form: This is a reality that makes no sense under the categories of spacetime. Thus, insofar as forms are shared as universals, they transcend spacetime.

    This whole paragraph is a good example of what I mean. You keep talking about “rectangular” as if it is some kind of thing. It is “shared”, “used”, “complete”, and so on. I fully agree that your use of these terms is appropriate, because we cannot help but talk this way by virtue of the way our language is, and I can even understand what you mean, at least if you are using those terms as analogous and metaphorical to how we interact with objects in the world. The problem is when you buy into the idea that all our words must refer to some kind of object, no matter how bizarre, then that you fall into all kinds of paradoxes and dilemmas. In this case, you are stuck trying to puzzle your way around how “rectangular” can interact between our minds and the world, which is just impossible.

    It would be better to just pay attention to our language and how it is used to better understand what we mean when we speak. To use a previous example, when I say, “no-one is coming”, that sentence shares the form of “John is coming”, and if I assumed that they operate according to the same rules, then I would be busy looking for “no-one”, and would be puzzled to be unable to find him. The solution is to see that superficial similarities mask deep differences in use, and that “no-one” is NOT like “John” at all. Similarly, “rectangular is shared between the mind and the world” looks like “an orange is shared between John and Jane”, but it is totally different, and you will drive yourself insane trying to look for “rectangular” somewhere and determining how it interacts with our minds and the world, and furthermore, will have to start coming up with a bizarre ontology where abstract concepts exist as substances both beyond space-time, but also somehow within it and interacting with it, which I find incredibly confusing.

    ReplyDelete
  125. dmt:

    >> So we can, if we choose, just stop at the fact that we use the word "red" to identify red things, and avoid all the problems and paradoxes that arise when we wonder how it is we are justified in attributing the single concept red to a variety of different things. Fair enough. But affirming that we can get along without addressing philosophical questions is not the same as answering them.

    Yes, but there is an assumption that these problems and paradoxes somehow are uncovering deeper issues that must be addressed. I suppose that if I held a particular view of language and reality, then I would drive myself crazy trying to find the entity nothing that “nothing” is supposed to refer to, and I would be distressed at the fact that the majority of people seem to be blind to this paradox of how something can be nothing and nothing something. However, an easier solution would be to see that this is not a problem at all, but a misunderstanding in how we actually use the word “nothing”. It does NOT refer to anything at all, let alone a mystical and mysterious Nothing.

    ReplyDelete
  126. "We see rectangular objects, and learn to call them “rectangular”, and learn the properties of “rectangular”, which allows us to use that term to refer to other rectangles that we come into contact with."

    So we have:
    1. Rectangular objects.
    2. The word "rectangular", that we learn to apply to rectangular objects.
    3. Learned properties of "rectangular."

    The fact that we learned to apply "rectangular" to certain objects, says nothing about whether we were justified in doing so or the relationship between our concept "rectangular" and the thing itself. Again, this is a philosophical question we don't have to ask, but it doesn't go away because we don't ask it.

    So when we learn the properties of "rectangle", we are learning the properties of "rectangle." There is nothing in the account that justifies us in applying the properties of "rectangle" beyond itself, other than our common habit of doing so. But, like I say, this simply avoids the philosophical question. What connects the properties of "rectangle" to something beyond the concept itself?

    ReplyDelete
  127. I think I can put the question succinctly:

    There are two objects in front of me, A1 and A2. I call A1 "rectangular" and A2 "triangular." What is it about A1 and A2, respectively, that justifies ascribing these terms to them? Note: I'm not asking the historical question of how it is we come to call them rectangular and triangular, I'm asking the philosophical question of why we are justified in doing so.

    ReplyDelete
  128. dguller,

    Time constraints mean I'm not going to be able to post again on this blog for a while (days).

    I really appreciate the respectful and thoughtful conversation, and wish you the best.

    David

    ReplyDelete
  129. dmt:

    >> There are two objects in front of me, A1 and A2. I call A1 "rectangular" and A2 "triangular." What is it about A1 and A2, respectively, that justifies ascribing these terms to them? Note: I'm not asking the historical question of how it is we come to call them rectangular and triangular, I'm asking the philosophical question of why we are justified in doing so.

    If A1 meets criteria for a rectangle, then it is a rectangle.

    If A2 meets criteria for a triangle, then it is a triangle.

    And the criteria are what we learn when we join the community of language users by interacting with rectangles and triangles, and being instructed in how to use the words “rectangle” and “triangle”.

    >> Time constraints mean I'm not going to be able to post again on this blog for a while (days). I really appreciate the respectful and thoughtful conversation, and wish you the best.

    No problem, and ditto. Take care.

    ReplyDelete
  130. dguller:

    I said:

    And when someone like dguller claims you should only accept things from empirical evidence, then atheism is out the window too, since the origin of the universe is in principle outside of the scope of observational science, due to the cosmological particle horizon. The default position in that case is agnosticism (literally "I don't know"), not atheism, and also not "agnostic atheism", since that position already attributes probabilities where a simple "I don't know" should stand in its place.

    You replied:

    Right. But you forget one important detail. The atheist does not stake his life upon what occurs outside of space-time. The theist does. The theist makes a variety of positive claims about how what happens outside of space-time is fundamentally important to how we live our lives within space-time, and claims to know how we should think, feel and behave on the basis of these speculations. It is the theist that is making the positive claim here, and all an atheist has to say is that the theist has no way of knowing whether what he or she believes is actually true.

    I am afraid you were missing my point. You appear to agree to what I said with "right", but then it is clear you don't, since you keep using the term atheist. My point was that under your own epistemological position you cannot remain an atheist, but must become an agnostic -- a true agnostic that is. "Agnostic atheist" won't do as I explained. And don't tell me that "the atheist does not stake his life upon what occurs outside of space-time". Sure s/he does. Granted, atheism can technically be defined as "lack of belief" but the vast majority of atheists are naturalists, and that seems to be the case with you too -- I have not seen evidence otherwise. Naturalism is a world view that makes positive claims, and cannot as such be defended under your own epistemological position.

    And even if you were not a naturalist, under atheism you assume, or least use as 'working hypothesis', that there is no God. You cannot do that under your own epistemological position, but must concede the fully unknowable -- which means being a true agnostic. Both the positions of agnostic atheist and of agnostic theist lean a certain way, and thus cannot be held under your own current epistemological position. Not that I believe that this position is correct, and I hope one day it will change for the better, I just point out where you have to be intellectually consistent and honest with yourself. Not just me, but several people by now have pointed out inconsistencies in your stance, including cl on another blog.

    And please don't come with the old, tired atheist standard reply that there is no real difference between agnosticism and agnostic atheism. True agnostics, with whom I had the pleasure to discuss and who are usually very well informed in philosophy, would vehemently disagree. Agnostic atheism weighs probabilities, agnosticism does not.

    ReplyDelete
  131. dguller:

    It is insufficient for the theist to then reply that the atheist does not know either. That does not automatically add warrant to the theist’s position at all. It just means that we are all in the same fog together and lack any clear compass to find our way.

    A more important question is whether one should base the foundations of one’s life upon beliefs that are utterly without foundation.


    One should not. However, naturalism contradicts what we know from science about actual matter and fields (before you reply, please read chapter 2 of my article, link above), while theism does not. Theism assumes something beyond the physical realm, which science cannot say anything about. And the assumption of that realm beyond the physical one is justified, among others, by classical philosophical arguments, by the cosmological fine-tuning argument and by the argument from reason in various forms (that argument, by the way, stands apart from the arguments from consciousness, which I personally do not care about). Furthermore, that assumption is supported by divine revelation. To me, neither philosophy nor divine revelation each on their own would be sufficient, but together they form a convincing combination of evidence for me. An analogy to this -- and I concede it is rather limited -- would be the convergence of paleontological and genetic evidence for evolution. Both of these are fine on their own, but together they make the evidence for evolution rock solid.

    I do think my beliefs have a sufficient foundation, a foundation that I find sorely lacking in naturalism.

    ReplyDelete
  132. Al:

    >> However, naturalism contradicts what we know from science about actual matter and fields (before you reply, please read chapter 2 of my article, link above), while theism does not.

    I tried to read it, but honestly, astrophysics is not my area of expertise, and won’t even pretend to understand everything of what you wrote.

    I will say one thing, though. You wrote there:

    “There are only two possibilities for the naturalist that can be taken seriously: the creation of the universe from eternal matter or from eternal fields. Some principle must have been there forever (again, it cannot simply ‘at some point’ have come out of nothing) and since the naturalist negates an eternal God, these are the logical choices.”

    I would add a third option. We do not know. Your dichotomy may be a false one. There may be other options that we do not know at this time. Furthermore, we do not know anything about eternity. All we know is what happened after the Big Bang, and what subsequently unfolded over space-time. Anything else is just speculation, which is not knowledge.

    And none of this contradictions naturalism or science.

    ReplyDelete
  133. I tried to post a longer text (already before my last post), but the post only briefly appears before vanishing again. I'll try it now in pieces.

    dguller:

    I said:

    And when someone like dguller claims you should only accept things from empirical evidence, then atheism is out the window too, since the origin of the universe is in principle outside of the scope of observational science, due to the cosmological particle horizon. The default position in that case is agnosticism (literally "I don't know"), not atheism, and also not "agnostic atheism", since that position already attributes probabilities where a simple "I don't know" should stand in its place.

    You replied:

    Right. But you forget one important detail. The atheist does not stake his life upon what occurs outside of space-time. The theist does. The theist makes a variety of positive claims about how what happens outside of space-time is fundamentally important to how we live our lives within space-time, and claims to know how we should think, feel and behave on the basis of these speculations. It is the theist that is making the positive claim here, and all an atheist has to say is that the theist has no way of knowing whether what he or she believes is actually true.

    I am afraid you were missing my point. You appear to agree to what I said with "right", but then it is clear you don't, since you keep using the term atheist.

    (cont.)

    ReplyDelete
  134. (cont.)


    My point was that under your own epistemological position you cannot remain an atheist, but must become an agnostic -- a true agnostic that is. "Agnostic atheist" won't do as I explained. And don't tell me that "the atheist does not stake his life upon what occurs outside of space-time". Sure s/he does. Granted, atheism can technically be defined as "lack of belief" but the vast majority of atheists are naturalists, and that seems to be the case with you too -- I have not seen evidence otherwise. Naturalism is a world view that makes positive claims, and cannot as such be defended under your own epistemological position.

    And even if you were not a naturalist, under atheism you assume, or least use as 'working hypothesis', that there is no God. You cannot do that under your own epistemological position, but must concede the fully unknowable -- which means being a true agnostic. Both the positions of agnostic atheist and of agnostic theist lean a certain way, and thus cannot be held under your own current epistemological position. Not that I believe that this position is correct, and I hope one day it will change for the better, I just point out where you have to be intellectually consistent and honest with yourself. Not just me, but several people by now have pointed out inconsistencies in your stance, including cl on another blog.

    (cont.)

    ReplyDelete
  135. Al:

    >> I am afraid you were missing my point. You appear to agree to what I said with "right", but then it is clear you don't, since you keep using the term atheist.

    First, your point was that for an atheist to be consistent with my principles, then he would have to be an agnostic at best, but not an atheist, because this implies having some type of knowledge, which I described as being impossible. I would say that someone can be inconsistent, but still be known by a certain label. People are inconsistent all the time, after all.

    Second, you are assuming that one cannot firmly negate a positive claim without having positive knowledge about the claim itself. That is not true. One can negate a positive position if it is logically incoherent, for example. One can negate a positive position if the positive reasons supporting it are fallacious. None of this is particularly controversial, I think.

    ReplyDelete
  136. Al:

    >> And even if you were not a naturalist, under atheism you assume, or least use as 'working hypothesis', that there is no God. You cannot do that under your own epistemological position, but must concede the fully unknowable -- which means being a true agnostic. Both the positions of agnostic atheist and of agnostic theist lean a certain way, and thus cannot be held under your own current epistemological position. Not that I believe that this position is correct, and I hope one day it will change for the better, I just point out where you have to be intellectually consistent and honest with yourself. Not just me, but several people by now have pointed out inconsistencies in your stance, including cl on another blog.

    And I have explained myself. I do not have to have knowledge about X to reject the claims of others who have weak justifications for the truth of X.

    ReplyDelete
  137. dugller,

    you are too fast pulling the trigger each time (which also makes me wonder how much you actually think about what others say before you reply). I said that this will be continued, but you interrupted the series (I had to wait a bit each time if my post will 'stick' in the system before posting the next part).

    I will post the last bit nonetheless, even in danger of you replying that you answered already (which I am not sure you did):

    And please don't come with the old atheist standard reply that there is no real difference between agnosticism and agnostic atheism. True agnostics, with whom I had the pleasure to discuss and who are usually very well informed in philosophy, would vehemently disagree. Agnostic atheism weighs probabilities, agnosticism does not.

    ReplyDelete
  138. Sorry for misspelling your name in the last post.

    ReplyDelete
  139. Al:

    >> And please don't come with the old atheist standard reply that there is no real difference between agnosticism and agnostic atheism. True agnostics, with whom I had the pleasure to discuss and who are usually very well informed in philosophy, would vehemently disagree. Agnostic atheism weighs probabilities, agnosticism does not.

    I won’t.

    Now, can you comment on what I wrote at March 10, 2011 8:24 AM? Maybe you will understand my position better.

    Thanks. And don’t worry about the misspelling.

    ReplyDelete
  140. dguller: "If A1 meets criteria for a rectangle, then it is a rectangle.

    If A2 meets criteria for a triangle, then it is a triangle.

    And the criteria are what we learn when we join the community of language users by interacting with rectangles and triangles, and being instructed in how to use the words “rectangle” and “triangle”."


    If I might take up where dmt117 left off (I probably won't do his arguments justice but here goes...)

    You seem to want to equate real properties: "triangularity", "redness", "rectangularity", etc with the language we use to describe those properties - as if the properties themselves don't exist or cease to exist if there is no one around to talk about them. I don't understand this line of reasoning. Could you explain this to me - how you arrive at this conclusion, (do I even have it right?)

    Thanks.

    ReplyDelete
  141. Daniel:

    >> You seem to want to equate real properties: "triangularity", "redness", "rectangularity", etc with the language we use to describe those properties - as if the properties themselves don't exist or cease to exist if there is no one around to talk about them. I don't understand this line of reasoning. Could you explain this to me - how you arrive at this conclusion, (do I even have it right?)

    Triangular objects, red objects, and rectangular objects do not depend on us for their existence. Our description of them as triangular, red and rectangular does depend upon our ability to perceive triangles, redness and rectangles, as well as upon the language that we use, because the concepts “triangular”, “red” and “rectangular” are rooted in our language.

    That’s all.

    ReplyDelete
  142. dguller,

    It turns out I don't have to do tonight what I thought I would, so I can get into it again...


    "If A1 meets criteria for a rectangle, then it is a rectangle.

    If A2 meets criteria for a triangle, then it is a triangle.

    And the criteria are what we learn when we join the community of language users by interacting with rectangles and triangles, and being instructed in how to use the words “rectangle” and “triangle”."

    Of course if A1 meets criteria for a rectangle, then it is a rectangle. This is just a restatement of the premise on which we both agree. We both agree that rectangles meet criteria to be rectangles. Where we differ is exactly what those criteria are and how A1 meets them, which was the point of the question.

    And, of course, we learned the criteria through our interactions with the community of language users. No one could possibly disagree with this. Nonetheless, we disagree, perhaps because my community of language users has long included Aristotelians and yours hasn't. An interesting historical point, perhaps, but of no philosophical relevance that I can see.

    I'm puzzled at the reluctance to simply talk straightforwardly about the what makes a rectangle a rectangle... especially as your approach is supposed to have the advantage over Aristotelianism of an admirable straightforwardness and simplicity. As an Aristotelian, I have no problem speaking straightforwardly about the properties of things, how they have them, and why we are justified in saying they have them.

    ReplyDelete
  143. Dmt:

    >> I'm puzzled at the reluctance to simply talk straightforwardly about the what makes a rectangle a rectangle... especially as your approach is supposed to have the advantage over Aristotelianism of an admirable straightforwardness and simplicity. As an Aristotelian, I have no problem speaking straightforwardly about the properties of things, how they have them, and why we are justified in saying they have them.

    I don’t think I was being reluctant. If you want more specifics, then I can say that a rectangle has four sides and a triangle has three sides. We can get into more specifics about the features of rectangles and triangles, too.

    I’m not too sure what else you want.

    ReplyDelete
  144. dguller,

    No, that's fine. The quality of four that you say the sides of a rectangle have: Is this the identically same four across rectangles, and is it the identically same four we have as a concept? I'm not talking about the sides of a particular rectangle, which are not identically the same across rectangles, but the quality of four that we both agree those sides possess.

    ReplyDelete
  145. dguller: "There was a time when I was devout and religious. It just never worked for me, and I simply lost my faith in a particular God, but still adhered to something higher and beyond, along the lines of deism, but even that was lost over time. It just became too hard to believe."

    Perhaps God is calling you back? Only this time it will be based on a firm foundation of real discovered truth rather than on tradition, upbringing, feelings or whatever else it may have been based on before.

    I was raised religious, but didn't really discover God until I came to him on my knees as a sinner in need of redemption. Thomism, which I've only recently discovered, just confirms the truth I already knew. Maybe you need the confirmation first?

    Let me leave you with this to ponder: The cause of this world cannot be something within the world - else it would be the cause of itself; a logical absurdity. So the cause of this world must be something not of this world. You don't need to extrapolate much beyond that.

    All the best to you in your search.

    ReplyDelete
  146. Dmt:

    >> The quality of four that you say the sides of a rectangle have: Is this the identically same four across rectangles, and is it the identically same four we have as a concept? I'm not talking about the sides of a particular rectangle, which are not identically the same across rectangles, but the quality of four that we both agree those sides possess.

    It is very tempting to talk about four as if it is some independent entity that is involved in both rectangles and our concept of “rectangle”. I would caution against this, because it leads to bizarre metaphysical conclusions. I would rather say that a rectangle has four sides, by definition, and so if we come across a shape that meets this criteria in the world, then we can call it a rectangle. I honestly do not know what else you need to make this work in our daily lives.

    ReplyDelete
  147. Daniel:

    Thanks for the kind words. I certainly have no ability to predict what is in store for me in the future. My life has already taken a number of twists and turns that my earlier self would never have anticipated. So, you never know.

    >> The cause of this world cannot be something within the world - else it would be the cause of itself; a logical absurdity. So the cause of this world must be something not of this world. You don't need to extrapolate much beyond that.

    I’ve just started Feser’s TLS, and then will read his Aquinas, and maybe Philosophy of Mind. After that, I will be in a better position to comment on what you said. Right now, what you said just seems wrong, but I’ll first let the good professor educate me on the full meaning of your claims.

    ReplyDelete
  148. Dmt:

    >> The quality of four that you say the sides of a rectangle have: Is this the identically same four across rectangles, and is it the identically same four we have as a concept? I'm not talking about the sides of a particular rectangle, which are not identically the same across rectangles, but the quality of four that we both agree those sides possess.

    It is very tempting to talk about four as if it is some independent entity that is involved in both rectangles and our concept of “rectangle”. I would caution against this, because it leads to bizarre metaphysical conclusions. I would rather say that a rectangle has four sides, by definition, and so if we come across a shape that meets this criteria in the world, then we can call it a rectangle. I honestly do not know what else you need to make this work in our daily lives.

    ReplyDelete
  149. dguller,

    I agree that we do not need to ask or answer this question - or any other philosophical question - to make things work in our daily lives. This is why Aristotle wrote that philosophy is the most noble but least necessary of endeavors, and why the perennial charge against philosophy since the time of Socrates is that it is useless.

    It does not follow, however, that because philosophical questions are not necessary to make things work, that they are not worth asking, or that they can't be answered, or even that the answers aren't valuable. Personally, I don't do philosophy to make things work; I make things work so I can do philosophy.

    As far as bizarre metaphysical conclusions go, they do not worry me. What worries me would be turning away from pursuit of the truth for fear that I might find it bizarre or, even worse, because I can't find a utilitarian purpose for it. If the truth is bizarre or useless, so be it. I pursue it for its own sake. In any case, I am quite familiar with the conclusions of Aristotelian and Thomistic philosophy, and I find them refreshingly lucid.

    Our difference then, seems to not be so much about metaphysics, but philosophy itself. If useless questions that may have bizarre answers are out of bounds for you, then we may not have much to discuss, because they are just the sort of questions I like to ask (which is why I haunt philosophy blogs like this one).

    ReplyDelete
  150. Dmt:

    >> It does not follow, however, that because philosophical questions are not necessary to make things work, that they are not worth asking, or that they can't be answered, or even that the answers aren't valuable.

    I agree with you, but you have to also realize that just because a question can be asked, it does not necessarily mean that it is meaningful.

    >> Personally, I don't do philosophy to make things work; I make things work so I can do philosophy.

    I don’t know what this means.

    >> As far as bizarre metaphysical conclusions go, they do not worry me. What worries me would be turning away from pursuit of the truth for fear that I might find it bizarre or, even worse, because I can't find a utilitarian purpose for it. If the truth is bizarre or useless, so be it. I pursue it for its own sake. In any case, I am quite familiar with the conclusions of Aristotelian and Thomistic philosophy, and I find them refreshingly lucid.

    That is fine. You are certainly free to do whatever you want with you life, and if holding bizarre and useless truths is something that you find valuable, then more power to you. With regards to my concerns about bizarre metaphysical beliefs, I would quote Feser: “the bizarreries and absurdities to which contemporary intellectuals have been led by their rejection of the four causes have been embraced as further surprising “discoveries” or “results” of philosophical inquiry, rather than recognized for what they are: a reductio ad absurdum of the premises laid down by their intellectual ancestors” (TLS p. 72). So, be careful that you do not embrace a philosophical position that results in bizarre beliefs, because that position may have been reduced to absurdity.

    >> Our difference then, seems to not be so much about metaphysics, but philosophy itself. If useless questions that may have bizarre answers are out of bounds for you, then we may not have much to discuss, because they are just the sort of questions I like to ask (which is why I haunt philosophy blogs like this one).

    That is fair. Anyway, thanks for the conversation. As you can see, I have begun TLS, and am enjoying it a great deal, although there are some points that I disagree with.

    ReplyDelete
  151. dguller,

    A question that is not meaningful is, as far as I'm concerned, not a question at all. e.g. What's the perimeter of a line?

    I accept bizarre truth because I accept truth absolutely. Large truth, small truth, common-sense truth, useless truth, mathematical truth, bizarre truth... the list of qualifiers is potentially endless. I'm interested in truth whatever its flavor. Perhaps if we have another conversation some day, I might try to convince you to accept truth unconditionally rather than restricting yourself to certain flavors of it.

    The absurd is false. Things that are bizarre are often absurd and therefore false, but not everything that is bizarre is absurd and false. Dr. Feser can speak for himself, but I suspect he rejects the doctrines of modern intellectuals because they false, not because they are bizarre. If not, then we part ways on this. (Anyone who rejects the bizarre because it is bizarre will find himself rejecting quantum mechanics.)

    Anyway, enjoyed the conversation and I wish you the best.

    ReplyDelete
  152. dguller:

    Now, can you comment on what I wrote at March 10, 2011 8:24 AM? Maybe you will understand my position better.

    I think I have answered this already with my post from March 11, 10:12 am. There is enough evidence, we do not start from something forever completely outside space-time. The evidence for God is accessible within our space-time from His divine revelation, and from the evidence that we see around us -- at the least the apparent design of our universe and features of the human mind that necessarily transcend physicalism, as I suggested in my post and in the background section of my cosmological article.

    I had said:
    There are only two possibilities for the naturalist that can be taken seriously: the creation of the universe from eternal matter or from eternal fields. Some principle must have been there forever (again, it cannot simply ‘at some point’ have come out of nothing) and since the naturalist negates an eternal God, these are the logical choices.

    You replied:
    I would add a third option. We do not know. Your dichotomy may be a false one. There may be other options that we do not know at this time. Furthermore, we do not know anything about eternity. All we know is what happened after the Big Bang, and what subsequently unfolded over space-time. Anything else is just speculation, which is not knowledge.

    And none of this contradictions naturalism or science.


    And what if the other option is God? That was my third option. And it is the one that, given the evidence in the post that I just mentioned, March 11, 10:12 am, appears the most plausible to me -- certainly far more plausible than an "option that we do not know at this time". I think you hide behind something 'science one day might uncover' -- while there is little reasonable expectation from what we *do* know that there might be something that we do not yet know which could explain a naturalistic origin of the universe. You stated earlier that you want to get to the truth. I am afraid that instead you are just putting up smokescreens and hide from the truth, because finding the truth might be uncomfortable.

    ReplyDelete
  153. Al:

    Does anything that is concluded by reason necessarily have to be true in reality?

    ReplyDelete
  154. Al:

    Does anything that is concluded by reason necessarily have to be true in reality?


    Certainly not, dguller. Also Feser is cautious about that. If I remember correctly, he says two or three times in Aquinas something to the effect of "if those premises (or argumentation) are true, then this and that follows by necessity". Yes, we can find the Thomistic arguments plausible or even convincing, but there is nothing like a mathematical or scientific proof for God.

    Also, I believe you have mentioned examples from science where what was concluded by reason turned out not to be the case in reality, and I support that. Science does not live from reason alone, but from checking against observation and experiment. As I write in my cosmological article:

    (http://home.earthlink.net/~almoritz/cosmological-arguments-god.htm)

    Indeed, as Alexander Vilenkin, one of the ‘fathers’ of quantum cosmology which deals with this kind of scenario, wrote in his paper Quantum Cosmology and Eternal Inflation: "sadly, quantum cosmology is not likely to become an observational science." But if it cannot become observational science, what kind of science is it then? The natural sciences are founded on observation and experiment. Shifting the foundations of the natural sciences towards an exploration of the world by pure thought alone would throw us back to – well, the pre-scientific world (even if the concepts and the mathematics are now more sophisticated).

    After all, as a scientist I dislike confusion of science with philosophy (while I love both).

    However, all this does not mean that we should throw our hands in the air and blindly follow scientism, the idea that scientific knowledge is the only real knowledge. If scientific knowledge was the only real knowledge, how could we believe anything? First of all, even science changes. For example, we had this beautiful edifice of Newtonian mechanics, and along came Einstein. Second, most of the knowledge that we go by in daily life is not scientifically tested knowledge. I do not, even cannot, scientifically test the idea that I can walk out the door to go to work and not be shot on the spot. I just assume it, as I assume thousands of things during any given day, as you do too. Third, if someone decides to marry his girlfriend, he cannot be scientifically certain that she truly loves him and will always keep on loving him. Yes, he can rigorously check for background and compatibility, but this cannot give absolute certainty. In the end it is about trust and taking the leap of faith that the marriage will be successful.

    In all these cases, we deal with knowledge and decisions based not on proof, but on plausibility.

    (cont.)

    ReplyDelete
  155. (cont.)

    Just like judging by plausibility is a fact of life in many areas, and daily life itself, as I just pointed out, also on the issue of theism vs. atheism it comes down to plausibility. And to me an unmoved mover that is a necessary being makes a whole lot more sense than a contingent wider universe that by chance exists as a perpetuum mobile that is just cranking out universes, for no other reason than that's just the way it is (our local universe cannot be all there is, it had a beginning with the Big Bang). Add to that the cosmological fine-tuning argument, against which all naturalistic arguments fail (see my article), and it is simply no contest. Also, to me philosophical arguments for a mind that goes beyond the brain make a lot more sense than the argument that the mind *is* the brain just because mental activity happens to correlate with brain activity (something that Aristotle and Aquinas would have predicted, as Feser points out, thus mind = brain on a correlation basis is a pseudo-scientific argument). Throw in divine revelation, and we arrive at a convergence of evidence that to me is highly plausible.

    Certainly far more plausible than atheism. I have pondered all the arguments for and against atheism far more thoroughly than the vast majority (though not all) atheists will ever do in their lifetime, and upon digging deeper, below the attractive surface, I found that the whole atheistic construct of naturalism rests on rather feeble foundations. My opinion of course.

    God left enough finger pointers in His creation *) and through divine revelation that I don't think scientism would be a good excuse not to take a leap of faith once the evidence is judged as being plausible. After all, faith is still required; if you had scientific proof, there would be certainty, not faith. If after death you discover that there is a God, but you say "you know what, I could never believe in you because I had no scientific proof' , I do not think God would appreciate that. He might view it as a pretty lame excuse, I am afraid.

    *) not just the cosmological fine-tuning argument and the argument from reason, but also the classical philosophical arguments start with observations from His creation

    ReplyDelete
  156. Al:

    How can reasoning based upon our experience of natural phenomena be able to bootstrap us out of the natural world into the realm beyond space-time? Just because it is highly effective in helping us to understand the natural world does not imply that it is equally effective beyond it. I guess that this is the core of my problem with this whole issue, because I think that just because something is implied by reason does not necessarily mean that it refers to something real. That is the core assumption behind natural theology and metaphysics, and I wonder why anyone would believe it.

    ReplyDelete
  157. dguller:

    How can reasoning based upon our experience of natural phenomena be able to bootstrap us out of the natural world into the realm beyond space-time? Just because it is highly effective in helping us to understand the natural world does not imply that it is equally effective beyond it. I guess that this is the core of my problem with this whole issue,

    First, remember what I had said in my post March 15, 7:24 pm:

    There is enough evidence, we do not start from something forever completely outside space-time. The evidence for God is accessible within our space-time from His divine revelation, and from the evidence that we see around us -- at the least the apparent design of our universe and features of the human mind that necessarily transcend physicalism, as I suggested in my post and in the background section of my cosmological article.

    See also my previous post.

    Second, not only is evidence for God accessible from within our space-time, we already know that certain things must be valid by necessity independent of our space-time. As Victor Reppert recently wrote:

    http://dangerousidea.blogspot.com/2011/03/naturalism-without-materialism.html

    Hiero5ant: You make the claim that all causal relations must involve spatio-temporal relations. This means, I take it, that anything that plays a causal role must have a particular location in space and time.

    Yet, if you draw a rational inference, you draw it in virtue of logical relations between the premises and the conclusion, and logical relations have no particular location in space and time, since they hold good regardless of spatiotemporal location. Yet those logical relationship are supposed to have an effect on what you conclude when you draw a logical conclusion. So I am not sure that this constraint is consistent with the possibility of rational inference.


    You might of course say, well, logical relations are only embodied in physical minds, and these are restrained to space-time. But this would be a tenuous argument. How can logical relations, if they are to be universally true, and ideas associated with them, 'jump' from mind to mind if they are physical (spatio-temporal) entities?

    because I think that just because something is implied by reason does not necessarily mean that it refers to something real.

    I have answered this in my previous post, outlining different kinds of knowledge. It is not clear to me why you keep repeating it.

    ReplyDelete
  158. Al:

    Thanks for your reply.

    >> The evidence for God is accessible within our space-time from His divine revelation, and from the evidence that we see around us -- at the least the apparent design of our universe and features of the human mind that necessarily transcend physicalism, as I suggested in my post and in the background section of my cosmological article.

    First, which divine revelation? There are many.

    Second, what aspects of the human mind “transcend physicalism”?

    >> How can logical relations, if they are to be universally true, and ideas associated with them, 'jump' from mind to mind if they are physical (spatio-temporal) entities?

    First, logical relations are abstracted from our experience in the world. We subsequently treat them as immutable and incorrigible, but they are rooted in our natural experience. When one is trying to decide whether a logical rule of inference works, what does one do? One tries it out with different premises and conclusions and sees if it leads to a false conclusion. All the examples in textbooks of logic are from our empirical experience.

    Second, they do not have to “jump” from mind to mind at all. We all have the same basic structure of our mind, and thus similar processes can be activated when we interact with one another. Why can’t logic be one of them? After all, we all live in the same world, which operates according to the same regularities and patterns, which we are able to reflect upon and abstract from to generate logical relationships. However, I fail to see that these logical relationships exist outside of space-time. I mean, just because we happen to treat them as indubitable and immutable does not mean that we are thereby touching beyond space-time.

    Again, all of this is taking our cognitive capacities to reach beyond space-time, which I find highly suspicious and grandiose. In the empirical world, we can test our rational deductions by testing them. Beyond the empirical world, we are stuck with the rational deductions themselves with no way of knowing whether they actually refer to anything real, except that they really, truly feel like they do. Is this feeling enough? What about a fiction writer who feels like they are contacting a real world in their fiction, populated by real individuals with real lives, and so on? Does that feeling of inspiration mean that they are really contacting some other realm?

    ReplyDelete
  159. @ dguller

    Though I strongly agree with you that the scientific method has been wonderfully successful at helping us to achieve a better understanding of the Universe(w.r.t. physical sciences especially), I would have to disagree with scientism's premise regarding scientific knowledge as being the only mode for achieving any sort of "real" knowledge of the world.

    Would you regard mathematics and logic (which you admit are methods for achieving knowledge distinct from the scientific method) as being the only methods, apart from the scientific method, which provide us with any real knowledge or do you happen to be aware of any others? I mention this because I'm curious as to your thoughts on Hayek's point regarding the limits of the scientific method in achieving an accurate description of the social orders (e.g. market economies in particular) which correspond to "real" phenomena, yet do not lend themselves to the usual methodology. Specifically his point mentioning the dangers inherent in misapplying said methods in achieving inaccurate, even erroneous descriptions of reality.

    http://nobelprize.org/nobel_prizes/economics/laureates/1974/hayek-lecture.html

    Admittedly Hayek is not arguing against the use of science in studying the economy, he's merely arguing against its misuse. So do you at least agree that the methods of physics are, say, different from the methods of biology, and those of the latter two are different to paleontology, archaeology and geology? Would you say that it's important to distinguish between the specific methodologies of these various scientific fields, lest we fail to obtain what we're looking for? So that there is no one scientific method but varying degrees of a method, some more promising at helping us to achieve knowledge than others?

    Lastly, math may be distinct from the more empirical sciences in helping us to achieve knowledge but I do not think it needs to eventually be verified through empirical science as you claim earlier. Godel & Tarski's metamath should be sufficient to dissuade anyone from believing this. Or merely contemplating Cantor's infinite sets will get you to see that the mind can achieve knowledge independent of the usual empirical sciences.

    Personally (and I'm no philosopher) I believe there are various methods for obtaining knowledge, some more suitable to certain scenarios than others. This is a meta-statement and I'm fine with that. I also believe that science is the most successful method humans have come up with for obtaining knowledge, though its reach is not allencompasing. I do believe that science will continue to astonish us with revealing more truth but that its approach to truth is asymptotic. Kinda like the onion Feynman once mentioned peeling away. The asymptote is "the way things really are" and science can only get closer, but never reach it for reasons Bronowski points out very clearly.

    Anyway, sorry if I rambled. I have much faith in science but am aware that it has limits in (and degrees of) applicability, and is incapable of providing us with answers to questions like: is there a God? For these specifically hard to get to topics I think intuition and imagination are good places to found our knowledge on. Math makes a lot of use of these too.

    ReplyDelete
  160. Hi Dr. Feser!

    I intend to read the entire series, so I can have a good grasp of the topic. (I also plan to do this with the other roundup posts you have on this blog. Thank you so much!)

    I have a question regarding the first article, "Blinded by Scientism," in the fifth to the last paragraph:
    ___

    Why is it that the "manifest image" and the "scientific image" of the [material] world "cannot be reconciled"? Is the idea that the [material] world is composed of "colorless, soundless, meaningless particles in motion" false, and that the explanations taught in high school about how color and sound work also wrong?

    Can qualitative sense perceptions never be accounted for with quantitative explanations? (Meaning and purpose definitely cannot be accounted for with measures, but what about color, temperature, and sounds?)
    ___

    Concerning the first question, I'm thinking what you mean is that the explanations for color, sound, etc. in terms of quarks does not convey the whole of reality, but only material reality. (Am I right? I'm sorry that I may have misunderstood the text.) With the second question, I don't have an idea...

    Thank you, Dr. Feser!

    ReplyDelete
  161. Dr Feser ,
    How do you respond to people who say that philosophy is dead , like Stephen Hawking ?
    Plus i heard Jerry Coyne accuse you of being dishonest by saying that the atheists who supposedly "refute" Aquinas's arguments are actually honest and know what Aquinas is saying and therefore they are not making caricatures.
    Saw it in a youtube channel called Rationality Rules who tried to attack Ben Shapiro and you in that interview you had with him.

    ReplyDelete